108
DESIGUALDADES Enfoque Problem-solving Gerard Romo Garrido

1 Las desigualdades fundamentales - Toomates · Segunda parte: Desigualdades geométricas. 10 Desigualdades con los lados del triángulo. → Desigualdad Triangular. La Transformación

  • Upload
    others

  • View
    3

  • Download
    0

Embed Size (px)

Citation preview

Page 1: 1 Las desigualdades fundamentales - Toomates · Segunda parte: Desigualdades geométricas. 10 Desigualdades con los lados del triángulo. → Desigualdad Triangular. La Transformación

DESIGUALDADES

Enfoque Problem-solving

Gerard Romo Garrido

Page 2: 1 Las desigualdades fundamentales - Toomates · Segunda parte: Desigualdades geométricas. 10 Desigualdades con los lados del triángulo. → Desigualdad Triangular. La Transformación

Toomates Coolección

Los documentos de Toomates son materiales digitales y gratuitos. Son digitales porque están pensados para ser consultados mediante un ordenador, tablet o móvil. Son gratuitos porque se ofrecen a la comunidad educativa sin coste alguno. Los libros de texto pueden ser digitales o

en papel, gratuitos o en venta, y ninguna de estas opciones es necesariamente mejor o peor que las otras. Es más: Suele suceder que los mejores

docentes son los que piden a sus alumnos la compra de un libro de texto en papel, esto es un hecho. Lo que no es aceptable, por inmoral y mezquino, es el modelo de las llamadas "licencias digitales" con las que las editoriales pretenden cobrar a los estudiantes, una y otra vez, por

acceder a los mismos contenidos (unos contenidos que, además, son de una bajísima calidad). Este modelo de negocio es miserable, pues

impide el compartir un mismo libro, incluso entre dos hermanos, pretende convertir a los estudiantes en un mercado cautivo, exige a los estudiantes y a las escuelas costosísimas líneas de Internet, pretende pervertir el conocimiento, que es algo social, público, convirtiéndolo en un

producto de propiedad privada, accesible solo a aquellos que se lo puedan permitir, y solo de una manera encapsulada, fragmentada,

impidiendo el derecho del alumno de poseer todo el libro, de acceder a todo el libro, de moverse libremente por todo el libro. Nadie puede pretender ser neutral ante esto: Mirar para otro lado y aceptar el modelo de licencias digitales es admitir un mundo más injusto, es

participar en la denegación del acceso al conocimiento a aquellos que no disponen de medios económicos, en un mundo en el que las modernas

tecnologías actuales permiten, por primera vez en la historia de la Humanidad, poder compartir el conocimiento sin coste alguno, con algo tan simple como es un archivo "pdf". El conocimiento no es una mercancía.

El proyecto Toomates tiene como objetivo la promoción y difusión entre el profesorado y el colectivo de estudiantes de unos materiales

didácticos libres, gratuitos y de calidad, que fuerce a las editoriales a competir ofreciendo alternativas de pago atractivas aumentando la calidad de unos libros de texto que actualmente son muy mediocres, y no mediante retorcidas técnicas comerciales.

Este documento se comparte bajo una licencia “Creative Commons”: Se permite, se promueve y se fomenta cualquier uso, reproducción y

edición de todos estos materiales siempre que sea sin ánimo de lucro y se cite su procedencia. Todos los documentos se ofrecen en dos versiones: En formato “pdf” para una cómoda lectura y en el formato “doc” de MSWord para permitir y facilitar su edición y generar versiones

parcial o totalmente modificadas. Se agradecerá cualquier observación, comentario o colaboración a [email protected]

Toomates Coolección consta de los siguientes libros:

Geometría axiomática:

Geometría Axiomática GA pdf 1 2 ... 23 portada

Problemas de Geometría PG pdf 1 2 3 4 5 6 7

Introducción a la Geometría PI pdf doc

Problem-solving:

Teoría de números AR pdf 1 2 3

Trigonometría PT pdf doc

Desigualdades DE pdf doc

Números complejos PC pdf doc

Álgebra PA pdf doc

Combinatoria PC pdf doc

Probabilidad PR pdf doc

Guía del estudiante de Olimpiadas Matemáticas OM pdf

Libros de texto (en catalán):

Àlgebra AG pdf 1 2

Funcions FU pdf doc

Geometria analítica GN pdf 1 2

Trigonometria TR

pdf doc

Nombres complexos CO pdf doc

Àlgebra Lineal 2n batxillerat AL pdf doc

Geometria Lineal 2n batxillerat GL pdf doc

Càlcul Infinitesimal 2n batxillerat CI pdf 1 2

Programació Lineal 2n batxillerat PL pdf doc

Recopilaciones de pruebas PAU España:

Catalunya TEC ST , Catalunya CCSS SC , Galicia SG

Recopilaciones de pruebas PAU Europa:

Portugal A SA, Portugal B SB

Recopilaciones de problemas olímpicos y preolímpicos:

IMO 1959-2019 SI, OME 1965-2019 SE, OMI 1997-2019 SD, AIME 1983-2020 SA

Cangur SR , Canguro SG , Kangourou SK

Versión de este documento: 10/07/2020

Todos estos documentos se actualizan constantemente. No uses una versión antigua, descarga la última versión de los documentos en los enlaces superiores.

www.toomates.net

Page 3: 1 Las desigualdades fundamentales - Toomates · Segunda parte: Desigualdades geométricas. 10 Desigualdades con los lados del triángulo. → Desigualdad Triangular. La Transformación

Índice Primera parte: Desigualdades algebraicas.

1 Las desigualdades fundamentales. →

2 La desigualdad AM-GM. →

3 El cuadro general de las desigualdades entre medias. →

4 La desigualdad Cauchy-Schwarz. →

5 El principio de reordenación. La desigualdad de Chebyshev. → El principio de reordenación de dos elementos. El principio de reordenación general.

La desigualdad de Chebyshev.

6 La desigualdad de Jensen. →

7 Desigualdades simétricas. Normalización y homogenización. →

8 Problemas olímpicos con desigualdades algebraicas. →

9 Desigualdades trigonométricas. →

Segunda parte: Desigualdades geométricas.

10 Desigualdades con los lados del triángulo. → Desigualdad Triangular. La Transformación de Ravi.

11 Desigualdades geométricas con trigonometría. →

12 Desigualdades con las rectas del triángulo. →

13 Inecuaciones. →

14 Aplicación de las desigualdades en la resolución de ecuaciones. →

Soluciones. →

Fuentes. →

Descarga de www.toomates.net/biblioteca/Olimpiadas.pdf

Una guía general de las Olimpiadas Matemáticas y sus contenidos curriculares.

Page 4: 1 Las desigualdades fundamentales - Toomates · Segunda parte: Desigualdades geométricas. 10 Desigualdades con los lados del triángulo. → Desigualdad Triangular. La Transformación
Page 5: 1 Las desigualdades fundamentales - Toomates · Segunda parte: Desigualdades geométricas. 10 Desigualdades con los lados del triángulo. → Desigualdad Triangular. La Transformación

Primera parte: Desigualdades algebraicas.

1 Las desigualdades fundamentales. 1.1 Las desigualdades fundamentales y sus aplicaciones.

Partimos de las dos desigualdades fundamentales:

a) 20 x y 00 2 xx

b) 22

2

2

1 ...0 nxxx y 0......0 21

22

2

2

1 nn xxxxxx

De estas dos anteriores se pueden deducir las dos siguientes: Si 0a y 0b ,

c) 222 baab

Sea bax . Entonces 2)(0 ba por el modelo (a), y por tanto:

22222 22)(0 baababbaba

d) 222 2)( baba

2)(0 ba , nuevamente por el modelo (a), y por tanto

222222222 22)(0 bababaabbaba

1.2

Demuestra que

a

b

b

a2

1.3

Demuestra que 2cossin

1

xx para

20

x

1.4 Demuestra que

aa

12

1.5

Demostrar que para todo x, se cumple:

21

2

2

2

x

x

Page 6: 1 Las desigualdades fundamentales - Toomates · Segunda parte: Desigualdades geométricas. 10 Desigualdades con los lados del triángulo. → Desigualdad Triangular. La Transformación
Page 7: 1 Las desigualdades fundamentales - Toomates · Segunda parte: Desigualdades geométricas. 10 Desigualdades con los lados del triángulo. → Desigualdad Triangular. La Transformación

2 La desigualdad AM-GM. 2.1 Desigualdad AM-GM con dos variables.

Si 0, ba

2

baab

y la igualdad solo sucede si ba .

Demostración. Hemos visto en 1.1c que 222 yxxy

Substituyendo a y b por a y b tenemos

222

baabbabaab

2.2 Desigualdad AM-GM en general.

La identidad anterior se puede generalizar para n números no negativos naa ,...1 :

n

aaaa nn

n

...... 1

1

y se produce la igualdad si y solo si naa ...1 .

2.3

Demostrar que si 0,, cba , entonces cbaaccbba 8))()((

2.4

Si 0ia y 1...21 naaa , entonces n

naaa 21...11 21

2.5

a) Demostrar que 222 cbaacbcab

b) Demostrar que 222233 cbacbaacbcab

2.6

Demuestra la desigualdad AM-GM con tres números: Si 0,, cba , entonces

33

cbacba

Page 8: 1 Las desigualdades fundamentales - Toomates · Segunda parte: Desigualdades geométricas. 10 Desigualdades con los lados del triángulo. → Desigualdad Triangular. La Transformación

2.7 Problema resuelto.

Sean naaa ,...,, 21 números reales positivos. Entonces, para cualquier permutación

nbbb ,...,, 21 de los mismos se cumple

nb

a

b

a

b

a

n

n ...2

2

1

1

Solución: En efecto, solo hay que observar que nn bbbaaa ...... 2121 , y por tanto,

aplicando la desigualdad AM-GM:

1...

......

21

212

2

1

1

n

n

nn

n

bbb

aaa

n

b

a

b

a

b

a

2.8

Sea 1n un número natural. Demuestra que n

nn

2

1!

2.9

Demuestra que si 6 zyx entonces 12222 zyx

2.10

Sean IRcba ,, , 1222 cba . Demuestra que 12

1

cabcab .

2.11

Demuestra que, si 0, ba , entonces 2

22 ba

ba

ba

.

2.12

Demostrar que si cba ,, son positivos, entonces:

abccbabcaacb ))()((

2.13

Si cba ,, son tres reales positivos cualesquiera, demostrar que

8

b

ac

a

cb

c

ba

2.14

Demuestra que si 0,, cba

222222222 9 cbacabcabaccbba

Page 9: 1 Las desigualdades fundamentales - Toomates · Segunda parte: Desigualdades geométricas. 10 Desigualdades con los lados del triángulo. → Desigualdad Triangular. La Transformación

2.15

Sean cba ,, números reales positivos tales que 1abc . Demuestra que

cbaa

c

c

b

b

a

2.16

Sean 0,,, yxba . Demuestra que abxybyaxxyab 4

2.17

Demostrar que, si 0,, cba , 222222)( baaccbcbaabc

2.18

Determina el mínimo de xx 22 cossin 44 .

2.19

Suponiendo que 0,, cba , demostrar que

cbaab

c

ac

b

bc

a

333

2.20

Sean cba ,, números reales positivos. Demostrar que

accbbacba 222333

2.21

Sean cba ,, números reales positivos. Demostrar que

accbbacabcabcba 222222333 2

2.22 D

Sean zyx ,, números reales positivos tales que 3 zyx . Demostrar que

3

1

2222

3

2

3

2

3

yx

z

xz

y

zy

x

2.23 D

Dados 0,, cba cumpliendo 1abc , demostrar que

3

cbac

ba

b

ac

a

cb

Page 10: 1 Las desigualdades fundamentales - Toomates · Segunda parte: Desigualdades geométricas. 10 Desigualdades con los lados del triángulo. → Desigualdad Triangular. La Transformación

2.24 M

Sean cba ,, números reales positivos tales que 3 cba . Demostrar que

cabcabcba

Russia MO 2004

Page 11: 1 Las desigualdades fundamentales - Toomates · Segunda parte: Desigualdades geométricas. 10 Desigualdades con los lados del triángulo. → Desigualdad Triangular. La Transformación

3 El cuadro general de las desigualdades entre medias.

La desigualdad AM-GM es un caso particular del cuadro general de desigualdades

"entre las medias".

3.1 Con dos variables:

bababa

baba

abba ,max

22

2,min

22

3.2 En general:

Si naa ,...1 son números no negativos:

QMAMGMHM

an

aa

n

aaaa

aa

na i

nnnn

n

i max......

...1

...1

min22

111

1

y se produce una igualdad si y solo si naa ...1 .

Nota: Las siglas se toman del inglés: HM: Harmonic-Mean, GM: Geometric Mean, AM: Arithmetic

Mean, QM: Quadratic Mean.

3.3

Dados dos números reales positivos yx, , se cumple )(2 yxyx

y la igualdad solo se cumple cuando yx .

3.4

Determina el valor máximo de

xx 722144

3.5 Problema resuelto.

Variaciones de la desigualdad AM-HM:

a) 2

1

1

1...

1... n

aaaa

n

n

b)

nn aa

n

aa

...

1...

1

1

2

1

y se cumple la igualdad si y solo si naa ...1

En efecto, partimos de la desigualdad AM-HM:

2

1

1

1

1 1...

1...

1...

1

...n

aaaa

aa

n

n

aa

n

n

n

n

Page 12: 1 Las desigualdades fundamentales - Toomates · Segunda parte: Desigualdades geométricas. 10 Desigualdades con los lados del triángulo. → Desigualdad Triangular. La Transformación

3.6 Problema resuelto. Desigualdad de Nesbitt (Inglaterra, 1903):

2

3

ba

c

ca

b

cb

a

Solución. Sea

cazcbybaxconzyx

zyx

cacbbacacbba

cacbbaccbbaa

cacbbacba

ba

bac

ca

cab

cb

cba

ba

ba

ca

ca

cb

cb

ba

c

ca

b

cb

a

ba

c

ca

b

cb

acbaf

,,3111

2

1

3111

2

1

3111

2

1

3111

3

3),,(

Primera demostración. Vemos que esta función se adapta perfectamente a la variación

de la desigualdad AM-HM anterior:

2

333

2

13

111

2

1),,( 2

zyxzyxcbaf

Segunda demostración. Aplicando la desigualdad demostrada anteriormente:

2

3),,(

361212216111

66111

),,(2

222

cbaf

y

z

z

y

x

z

z

x

x

y

y

x

z

z

y

z

x

z

z

y

y

y

x

y

z

x

y

x

x

x

zyxzyxcbaf

Tercera demostración. Aplicando la desigualdad AM-GM:

33 33

zyxzyxzyxzyx

La aplicamos a los dos paréntesis:

333

13

13

1113

111

zyxzyxzyxzyx

2

33

2

9133

2

13

111

2

1),,(

3

3

zyxzyx

zyxzyxcbaf

Page 13: 1 Las desigualdades fundamentales - Toomates · Segunda parte: Desigualdades geométricas. 10 Desigualdades con los lados del triángulo. → Desigualdad Triangular. La Transformación

4 La desigualdad Cauchy-Schwarz.

4.1 Desigualdad Cauchy-Schwarz.

Para cualquier conjunto de números reales naaa ,...,, 21 y nbbb ,...,, 21 ,

22

2

2

1

22

2

2

1

2

2211 ......... nnnn bbbaaabababa

y la igualdad solo pasa cuando las n-tuplas son proporcionales: n

n

b

a

b

a

b

a ...

2

2

1

1

4.2 Corolario.

22

2

2

1

2

21 ......

nn aaa

n

aaa

Demostración.

Basta tomar 1...21 nbbb .

4.3 Desigualdad Cauchy-Schwarz "en la forma de Engel".

Para cualquier conjunto de números reales naaa ,...,, 21 y para cualquier conjunto

nbbb ,...,, 21 de números positivos,

n

n

nn bbb

b

a

b

a

b

aaaa

......... 21

2

2

2

2

1

2

12

21

Demostración.

Basta aplicar la desigualdad Cauchy-Schwarz con i

ii

b

aa ' y ii bb ' .

4.4 Corolario a la Desigualdad Cauchy-Schwarz "en la forma de Engel".

Para cualquier conjunto nbbb ,...,, 21 de números reales positivos,

n

nbbb

bbbn1

...11

...21

21

2

y la igualdad acontece si y solo si nbbb ...21 .

Demostración.

Basta tomar 1...21 naaa

Alternativamente, se puede demostrar mediante la desigualdad AM-GM:

2

21

21

2121

2121

1...

11...1

...111

...11

......

nbbb

bbb

bbbn

bbb

bbbnbbb

n

nn

nn

nnn

Page 14: 1 Las desigualdades fundamentales - Toomates · Segunda parte: Desigualdades geométricas. 10 Desigualdades con los lados del triángulo. → Desigualdad Triangular. La Transformación

Nota histórica.

Bunyakovskii (1804-1889) publicó esta desigualdad en una monografía sobre

desigualdades entre integrales en 1859, veinticinco años antes que Schwarz (1843-

1921), pero es más conocida como desigualdad de Cauchy-Schwarz.

4.5 Demostrar que la desigualdad AM-QM es un caso particular de la desigualdad Cauchy-

Schwarz.

4.6

Sean zyx ,, números reales positivos tales que 1 zyx . Determina el valor mínimo

de zyx

941 .

4.7

Encontrar el máximo de la función xbxaxf cossin)( , con 0, ba , y 2/0 x .

4.8

Sean edcba ,,,, números reales tales que 8 edcba y

1622222 edcba

Hallar el valor máximo de e.

4.9

Demostrar que si 3 cba , entonces 3222 cba

4.10

Demostrar que si 0,, cba , entonces 222222222 9 cbacabcabaccbba

4.11

Demuestra que si 0,,, dcba , entonces dcbadcba

6416411

4.12 M

Sean cba ,, números reales positivos. Demostrar que si

1111 2

2

2

2

2

2

c

c

b

b

a

a

Entonces 2

3 cabcab .

Page 15: 1 Las desigualdades fundamentales - Toomates · Segunda parte: Desigualdades geométricas. 10 Desigualdades con los lados del triángulo. → Desigualdad Triangular. La Transformación

4.13 F

Demuestra el siguiente corolario a la desigualdad Cauchy-Schwarz:

221

2

21

2

2

2

2

2

1

2

1 bbaababa

y acontece la igualdad si y solo si las parejas ),( 21 aa y ),( 21 bb son proporcionales.

Que se puede generalizar por inducción al caso de n números:

21

2

1

222

2

2

2

2

1

2

1 ......... nnnn bbaabababa

y acontece la igualdad si y solo si las tuplas ),...,( 1 naa y ),...,( 1 nbb son proporcionales.

4.14 MD

Sean 0,, cba . Demostrar que

0222 222

2

222

2

222

2

bac

abc

acb

cab

cba

bca

Pham Kim Hung

4.15 F

Supongamos que 1,, zyx y 2111

zyx. Demostrar que

111 zyxzyx

IRAN MO 1998

4.16 F

Sean x, y, z reales positivos tales que 3 zyx . Halla el valor máximo

alcanzado por

6322 zyx

¿Para qué valores de x, y, z se alcanza dicho máximo?

OME 2015 Primera fase, segunda sesión #6

Page 16: 1 Las desigualdades fundamentales - Toomates · Segunda parte: Desigualdades geométricas. 10 Desigualdades con los lados del triángulo. → Desigualdad Triangular. La Transformación
Page 17: 1 Las desigualdades fundamentales - Toomates · Segunda parte: Desigualdades geométricas. 10 Desigualdades con los lados del triángulo. → Desigualdad Triangular. La Transformación

5 El principio de reordenación. La desigualdad de Chebyshev.

El principio de reordenación de dos elementos.

5.1

MF

Demuestra que, si ba y yx , entonces bxaybyax .

A esta desigualdad la llamaremos "Principio de Reordenación de dos elementos".

5.2 MF

Demuestra que, si 0, yx ,

yxx

y

y

x

22

5.3

F

Sean dcba ,,, números reales tales que cbda . Demostrar que

0))(())(())(( cbaddbcadcba

Czech and Slovak Republics, 2004

El principio de reordenación general. El Principio de reordenación de dos elementos introducido en el apartado anterior se

puede extender a cualquier n-tupla de números:

Dadas dos secuencias ordenadas de números: naaa ...21 y nbbb ...21 ,

La suma nnbababa ...2211 es maximal, la suma nnn bababa 1211 ... es minimal.

Esto quiere decir que la suma con cualquier otra permutación naaa ',...,',' 21 se

encontrará entre estas dos:

nnnnnnn bababababababababa ...'...''... 221122111211

Además, se cumple

nnnnnnn aaaaaababababababa ,...,,',...,',''...''... 212122111211

y se cumple

112122112211 ,...,,',...,','...'...'' aaaaaababababababa nnnnnnn

Nota:

Para simplificar la escritura, en las primeras soluciones de este tema se utiliza la

siguiente notación:

332211

321

321bababa

bbb

aaa

Page 18: 1 Las desigualdades fundamentales - Toomates · Segunda parte: Desigualdades geométricas. 10 Desigualdades con los lados del triángulo. → Desigualdad Triangular. La Transformación

5.4 D

Demostrar la desigualdad AM-GM aplicando el Principio de Reordenación.

5.5

Demostrar que acbcabcba 222

5.6

Demostrar que 222333 cbbaaccba

Nota: Este problema ya fue propuesto en 2.20. Se propone ahora resolverlo mediante reordenación.

5.7

Demostrar la desigualdad de Nesbitt introducida en 3.6 aplicando la técnica de este

apartado.

5.8

Demostrar que, dados naa ,...,1 números reales positivos, y naas ...1 , se cumple:

1...

2

2

1

1

n

n

as

a

as

a

as

a

n

n

5.9

Demostrar que si 0,, cba , entonces

cbaba

abc

ca

acb

cb

bca

222

Nota: Los problemas 10.12 y 10.15 de desigualdades con los lados del triángulo están

resueltos mediante el principio de reordenación.

Page 19: 1 Las desigualdades fundamentales - Toomates · Segunda parte: Desigualdades geométricas. 10 Desigualdades con los lados del triángulo. → Desigualdad Triangular. La Transformación

La desigualdad de Chebyshev.

Versión directa de la desigualdad de Chebyshev.

Sean dos secuencias ia y ib ordenadas de la misma forma (es decir, ambas

crecientes o ambas decrecientes). Entonces, aplicando el principio de la reordenación

que acabamos de ver, tenemos:

112111

2423111

1322111

221111

......

...........................................................

......

......

......

nnnnn

nnn

nnn

nnnn

bababababa

bababababa

bababababa

bababababa

Sumando todas estas desigualdades obtenemos:

nnnn bbaababan ......... 1111

Versión inversa de la desigualdad de Chebyshev.

Si las secuencias ia y ib están ordenados de forma inversa, es decir, una es creciente

y la otra decreciente, tenemos la desigualdad contraria:

nnnn babanbbaa ......... 1111

5.10 Problema resuelto.

Demostrar que, si 0,, cba ,

1888 222

abc

c

cab

b

bca

a

IMO 2001 #2

Solución:

Aplicando 222 yxxy ,

bcSbccbabcbcabca 66628 22222

donde 222 cbaS . Luego

abS

c

acS

b

bcS

a

abc

c

cab

b

bca

a

666888 222

Vemos que las secuencias

),,( cba y

abSacSbcS 6

1,

6

1,

6

1

están ordenadas de la misma forma, y por tanto podemos aplicar la desigualdad de

Chebyshev:

Page 20: 1 Las desigualdades fundamentales - Toomates · Segunda parte: Desigualdades geométricas. 10 Desigualdades con los lados del triángulo. → Desigualdad Triangular. La Transformación

abSacSbcScba

abS

c

acS

b

bcS

a

6

1

6

1

6

1

3

1

666

Ahora aplicamos la desigualdad AM-HM (ver 3.5b):

abSacSbcSabSacSbcS 666

9

6

1

6

1

6

1

Y aplicamos la desigualdad QM-AM:

(*)

666

3

666

3

abSacSbcSabSacSbcS

Finalmente observamos que

23)(23666 cbaacbcabSabSacSbcS

Y por tanto

cbacba

1

3

3(*)

2

Con lo que llegamos a:

11

666

cbacba

abS

c

acS

b

bcS

a

Tal y como queríamos ver. La igualdad se cumple cuando 1 cba .

Nota: Este mismo problema se volverá a resolver en 6.4 mediante otra técnica.

Fuente: https://www.artofproblemsolving.com/Wiki/index.php/2001_IMO_Problems/Problem_2

5.11 F

Sean dcba ,,, números reales positivos tales que 42222 dcba . Demostrar que

3

42222

cba

d

bad

c

adc

b

dcb

a

Page 21: 1 Las desigualdades fundamentales - Toomates · Segunda parte: Desigualdades geométricas. 10 Desigualdades con los lados del triángulo. → Desigualdad Triangular. La Transformación

6 La desigualdad de Jensen.

6.1 Definición. Función convexa.

Diremos que una función )(xf es convexa en ba, cuando, para cualquier par yx

en ba, se cumpla

ytxtfyftxft )1()1(

es decir, cuando la gráfica de la función está siempre por debajo del segmento que une

dos de sus puntos.

6.2 Teorema.

Si 0)('' xf en ba, , entonces la función es convexa en ba, .

Demostración. Tomamos valores byxa y 10 t . Fijando los valores ty, ,

consideremos la función

yttxfyftxtfxg )1()()1()()( .

Entonces

yttxfxftyttxftxtfxg )1(')(')1(')(')('

Por hipótesis, 0)('' xf ,y por tanto )(' xf es creciente. Luego:

0)('

0)1(')('

0)1(')('

)1(')('

)1(

xg

yttxfxft

yttxfxf

yttxfxf

yttxx

Es decir, la función )(xg es decreciente, y por tanto, puesto que en todo momento

estamos suponiendo yx , tenemos que 0)()( ygxg .

Finalmente:

yttxfyftxtf

yttxfyftxtfxg

)1()()1()(

)1()()1()()(0

tal y como queríamos ver.

Page 22: 1 Las desigualdades fundamentales - Toomates · Segunda parte: Desigualdades geométricas. 10 Desigualdades con los lados del triángulo. → Desigualdad Triangular. La Transformación

6.3 Teorema. Desigualdad de Jensen.

Si f es una función convexa en IRba , , entonces, para todo baxxx n ,...,,, 21 , y

para todo 0...,,, 21 naaa cumpliendo 1...21 naaa , se cumple

nnnn xaxaxafxfaxfaxfa ...)(...)()( 22112211

En particular, tomando naaa n /1...21 , tenemos el siguiente corolario:

Si f es una función convexa en IRba , , entonces, para todo baxxx n ,...,,, 21 ,

n

xxxfnxfxfxf n

n

...)(...)()( 21

21

Nota: Si la función es cóncava se verifican las desigualdades contrarias.

6.4 Problema resuelto.

Demostrar que, si 0,, cba ,

1888 222

abc

c

cab

b

bca

a

IMO 2001 #2

Solución:

Primera versión.

Puesto que la desigualdad es homogénea, podemos suponer 1 cba (ver Tema 7).

La función x

xf1

)( es convexa, luego podemos aplicar la desigualdad de Jersen:

(*))8()8()8(

1

8

1

8

1

8

1222222 abcccabbbcaaabc

ccab

bbca

a

Y ahora tenemos en cuenta que 1 cba , y por tanto

abccbacacbbacba

abccacbbaabccba

bcaccbabcabaabccbacba

24))()((3

2))()((36

361

333333

333

2222223333

En donde hemos tenido en cuenta abccacbba 8))()(( (ver problema 2.3)

Finalmente: 11

)(

1(*)24

3

3333

cbacba

cbaabccba

Segunda versión.

De nuevo, puesto que la desigualdad es homogénea, podemos suponer 1 cba (ver

Tema 7).

c

abcc

c

b

abcb

b

a

abca

a

abc

c

cab

b

bca

a

88888 222

222

Page 23: 1 Las desigualdades fundamentales - Toomates · Segunda parte: Desigualdades geométricas. 10 Desigualdades con los lados del triángulo. → Desigualdad Triangular. La Transformación

abcc

c

abcb

b

abca

a

c

abcc

c

b

abcb

b

a

abca

a

c

abcc

c

b

abcb

b

a

abca

a

888

88888

3

3

3

3

3

3

2

2

2

2

2

2

2

2

2

2

2

2

La función abcx

xxf

8)(

3

3

es cóncava, y por tanto podemos aplicar la desigualdad

de Jersen:

cbafcfbfaf

3

1

3

1

3

13)()()(

Aplicando la desigualdad AM-GM, y teniendo en cuenta que la función )(xf es

estrictamente creciente:

13

133

3

1

3

1

3

13

3

1

3

1

3

1 33

abcfcbafabccba

Fuente: https://www.artofproblemsolving.com/Wiki/index.php/2001_IMO_Problems/Problem_2

6.5

Dados cba ,, números reales no negativos tales que 1 cba , demostrar que

3

1 accbba

6.6

Demuestra la desigualdad AM-GM como caso particular de la desigualdad de Jensen.

6.7

Si 0,, cba y 1 cba , determina el mínimo de 101010

111

cc

bb

aa

6.8

Demostrar que, en todo triángulo ABC ,

2

33sinsinsin CBA

es decir, el mínimo perímetro de un triángulo inscrito en una circunferencia fija se

obtiene con el triángulo equilátero.

Page 24: 1 Las desigualdades fundamentales - Toomates · Segunda parte: Desigualdades geométricas. 10 Desigualdades con los lados del triángulo. → Desigualdad Triangular. La Transformación

6.9

Demostrar que, en todo triángulo ABC ,

CBA tantantan33

6.10

Demostrar que, en todo triángulo ABC ,

2tan

2tan

2tan3

CBA

6.11 D

Suponiendo que 1,2/1,...,, 21 naaa , demostrar que

nn

n

n

n

n

aaan

aaa

aaa

aaa

...

)1)...(1)(1(

...

...

21

21

21

21

Las tres proposiciones siguientes nos permiten ahorrarnos verificar la convexidad de la función:

6.12 Proposición.

Si una función real IRbaf ,: satisface la condición

22)()(,,

yxfyfxfbayx

Entonces, para todo baxxx n ,...,,, 21 , se cumple

n

xxxfnxfxfxf n

n

...)(...)()( 21

21

6.13 Proposición.

Si una función real IRbaf ,: satisface la condición

yxfyfxfbayx 2)()(,,

Entonces, para todo baxxx n ,...,,, 21 , se cumple

nnn xxxfnxfxfxf ...)(...)()( 2121

Observación. La proposición anterior se puede generalizar a cualquier tipo de media: Aritmética, geométrica,

armónica...

6.14 Proposición.

Si f es una función definida en IRba , , entonces, para todo baxxx n ,...,,, 21 , y

para todo 0...,,, 21 naaa cumpliendo 1...21 naaa , se cumple

nnnn xaxaxafxfaxfaxfa ...)(...)()( 22112211

si y solo si se cumple para el caso 2n .

Page 25: 1 Las desigualdades fundamentales - Toomates · Segunda parte: Desigualdades geométricas. 10 Desigualdades con los lados del triángulo. → Desigualdad Triangular. La Transformación

6.15 Problema resuelto.

Supongamos que 1,...,, 21 nxxx . Demostrar que

nnn xxx

n

xxx ...11

1...

1

1

1

1

2121

IMO Shortlist

Solución: Aplicando 6.13, es suficiente demostrar que, para todo 1, yx ,

yxyx

1

2

1

1

1

1

Que es equivalente a demostrar

yxyx

1

2

1

1

1

122

0)1()(

02222

222222

11212

1

2

11

2

1

2

1

1

1

1

2

332222222233

22223322

2222

22

22

22

xyyx

xyxyyxyxyxyxyxxyxyyx

yxyxxyxyyxyx

yxyxxy

yxyx

xy

yxyx

Lo cual es cierto, pues 0)1()(0111, 2 xyyxxyxyyx

Page 26: 1 Las desigualdades fundamentales - Toomates · Segunda parte: Desigualdades geométricas. 10 Desigualdades con los lados del triángulo. → Desigualdad Triangular. La Transformación
Page 27: 1 Las desigualdades fundamentales - Toomates · Segunda parte: Desigualdades geométricas. 10 Desigualdades con los lados del triángulo. → Desigualdad Triangular. La Transformación

7 Desigualdades simétricas. Normalización y homogenización.

7.1 Definición. Desigualdades simétricas.

Una desigualdad simétrica es aquella que se puede expresar como

0,...,, 121 aaaf

Cumpliendo 121 ,...,,,...,,21

aaafaaafniii para cualquier permutación niii ,,..., 21

de n,...,2,1 .

Un ejemplo clásico de desigualdad simétrica es la Desigualdad de Schur:

7.2 Problema resuelto. Desigualdad de Schur.

Suponiendo 0,, cba , entonces

)()()(3333 accacbbcbaababccba

Solución.

0))(())(())((

0)()()(3

)()()(3

333

333

bcaccabcbbcabaa

accacbbcbaababccba

accacbbcbaababccba

Aprovechando que la desigualdad es simétrica, podemos suponer que cba .

Sean bax , cby . Entonces:

cyxacacbbayx

cyb

y la desigualdad queda:

)2()()()())((

)))((()()()()(0

222 yxxyxyxcyxcycyxyyxcyxx

yyxcxycyyxxcyx

Lo cual es cierto porque 0,, cyc . La igualdad acontece cuando 0 yx y 0 cx ,

es decir, cuando cba o cuando 0, cba (o cualquiera de sus permutaciones).

7.3 Observación.

Esta desigualdad es equivalente a

abcbacacbcba ))()(( con 0,, cba ,

que fue demostrada independientemente en el problema 2.12.

7.4 Desigualdad de Schur generalizada.

Si 0,, cba y tomando cualquier constante 0k , se cumple:

0))(())(())(( bcacccbabbcabaa kkk

7.5 Observación.

Se puede demostrar que la desigualdad de Schur es cierta también para 0k . Y se

puede demostrar que si k es par, la desigualdad es cierta para cualquier terna cba ,, no

necesariamente positivos.

Page 28: 1 Las desigualdades fundamentales - Toomates · Segunda parte: Desigualdades geométricas. 10 Desigualdades con los lados del triángulo. → Desigualdad Triangular. La Transformación

Homogenización.

7.6 Definición. Inecuaciones homogéneas.

Decimos que una función ,...),,( cbaf es homogénea de grado n si, para todo 0k ,

,...),,(,...),,( cbafkkckbkaf n

En particular, diremos que es homogénea de grado 0 si ,...),,(,...),,( cbafkckbkaf

Por ejemplo, la función bca

acbaf

8),,(

2 con 0,,, cba es homogénea de grado 0

puesto que

),,(

888),,(

222cbaf

bca

a

bcak

ka

kbkcka

kakckbkaf

Llamaremos desigualdad homogénea de grado n a toda desigualdad de la forma

0,...),,( cbaf con ,...),,( cbaf homogénea de grado n.

Las desigualdades homogéneas de grado 1 se pueden "escalar", es decir, podemos

multiplicar sus variables por cualquier valor de 0k . En efecto:

,...),,(0,...),,(,...),,(0 ckbkakfcbafkcbaf

Homogenizar una inecuación es utilizar la condición dada en el enunciado convertirla

en una inecuación homogénea equivalente.

7.7 F

Si 0,, cba y 1 cba , demostrar que

cabcabcba 41222

7.8 M

Sean cba ,, números reales positivos tales que 1abc . Demostrar que

cbaa

c

c

b

b

a

7.9 M

Sean cba ,, números reales positivos tales que 1abc . Demostrar que 222 cbacba

Page 29: 1 Las desigualdades fundamentales - Toomates · Segunda parte: Desigualdades geométricas. 10 Desigualdades con los lados del triángulo. → Desigualdad Triangular. La Transformación

7.10 Definición. Normalización.

Las desigualdades homogéneas se pueden normalizar, es decir, podemos añadir

restricciones que no figuraban en el enunciado.

Por ejemplo, supongamos que queremos demostrar 03333 abccba . Aunque no

figura en el enunciado, podemos añadir la condición 1abc . En efecto, supongamos

que 3kabc , y sean 'kaa , 'kbb , 'kcc .

Entonces 1''' cba , y nuestra desigualdad se convierte en 0'''3''' 333 cbacba , que es

la misma que la anterior.

7.11 M

Sean cba ,, números positivos. Demostrar que

abcabcacabccbabcba

1111333333

7.12

Demostrar que, si 0,, cba ,

1888 222

abc

c

cab

b

bca

a

IMO 2001 #2

7.13 Observación.

Una lista ampliada de condiciones que se pueden añadir a una desigualdad homogénea

podría ser la siguiente:

a) 1a b) 1b c) 1c d) 1abc

e) 1 cabcab f) 1222 cba

7.14 F

Sean cba ,, números reales positivos tales que 1abc . Demuestra que

11

11

11

1

ac

cb

ba

IMO 2000 #2

Page 30: 1 Las desigualdades fundamentales - Toomates · Segunda parte: Desigualdades geométricas. 10 Desigualdades con los lados del triángulo. → Desigualdad Triangular. La Transformación

8 Problemas olímpicos con desigualdades algebraicas.

8.1 F

a) Demuestra que

10

1

100

99...

8

7

6

5

4

3

2

1

b) Demuestra, por inducción, que

113

1

2

12...

8

7

6

5

4

3

2

1

n

nn

n

c) Con ayuda del apartado anterior, mejora la desigualdad del apartado (a) demostrando

que

12

1

100

99...

8

7

6

5

4

3

2

1

8.2 MF

Demostrar que si 0,,, dcba , entonces

3333316 dcbadcba

8.3 F

Supongamos que 1 cba , siendo 0,, cba . Demostrar que

6191919 cba

8.4 MF

Sean cba ,, números positivos. Demostrar que

222

111

cbaabc

cba

8.5 M

Sean cba ,, números reales positivos tales que 1abc . Prueba la desigualdad siguiente:

4

3

111

222

ac

c

bc

b

ab

a

OME Fase Nacional 2009 #5

Page 31: 1 Las desigualdades fundamentales - Toomates · Segunda parte: Desigualdades geométricas. 10 Desigualdades con los lados del triángulo. → Desigualdad Triangular. La Transformación

8.6 MD

Sean 0,, cba cumpliendo

4222 abccba

demostrar que

20 abccabcab

USAMO 2001 #3

8.7 M

Sean cba ,, números reales positivos tales que

42222 cbacba

Demostrar que

3

111222

ac

ca

cb

bc

ba

ab

USAMO 2011 #1

8.8 D

Dados 0,, cba , demostrar que

82

2

2

2

2

222

2

22

2

22

2

bac

bac

acb

acb

cba

cba

USAMO 2003 #5

8.9 F

Dados naaa ...,,, 21 enteros positivos diferentes, demostrar que

n

k

n

k

k

kk

a

112

1

IMO 1978 #5

8.10 F

Demuestra que

b

baba

ba

a

ba

8

)(

28

)( 22

para todo 0 ba .

Page 32: 1 Las desigualdades fundamentales - Toomates · Segunda parte: Desigualdades geométricas. 10 Desigualdades con los lados del triángulo. → Desigualdad Triangular. La Transformación

8.11 F

Sean a, b y n enteros positivos tales que ba y 21 nab . Prueba que

34 nba .

Indica justificadamente cuando se alcanza la igualdad.

OME 2013 Fase nacional #1

8.12 F

Sean ba, números positivos. Probar que

2

22 baabba

OME 2014 Fase local, segunda sesión #4

8.13 D

Sean cba ,, tres números reales positivos. Demuestra que

8

15

332

3

323

3

233

3

cba

cba

cba

cba

cba

cba

OME 2010 Fase nacional, segunda sesión #4

8.14 F

Demuestra que

222byaxbyax

para cualesquiera IRyx , y cualesquiera IRba , con 1 ba , 0, ba .

¿En qué casos se da la igualdad?

OME 2015 Primera Fase, primera sesión #1

8.15 F

Dados dos números reales positivos p, q tales que 1 qp , y sabiendo que todo par de

números reales yx, cumple 02 yx , se pide demostrar

a) xyyx

2

b)

222

22

yxyx

c) 2

251122

qq

pp

OME 1984 Fase nacional #3

Page 33: 1 Las desigualdades fundamentales - Toomates · Segunda parte: Desigualdades geométricas. 10 Desigualdades con los lados del triángulo. → Desigualdad Triangular. La Transformación

8.16 MD

Sean x e y números reales entre 0 y 1. Probar que

yxxyxxyxyx 2223 22

OME 2014 Primera fase, segunda sesión #5

8.17 MD

Prueba que para cualesquiera números reales a,b tales que 1,0 ba , se cumple la

desigualdad siguiente:

2)1()1()1)(1( 2222 bababaab

OME 2008 Fase nacional #2

8.18 D

a) Demostrar que

1)1()1()1( 2

2

2

2

2

2

z

z

y

y

x

x

Para todos los números reales zyx ,, , todos diferentes de 1 y cumpliendo 1xyz .

b) Demostrar que la igualdad acontece para infinitas ternas de números racionales

zyx ,, , todos diferentes de 1 y cumpliendo 1xyz .

IMO 2008 #2

Nota: Solo el apartado (a) es un problema de desigualdades. El apartado (b) es un

problema propio de Teoría de Números.

Page 34: 1 Las desigualdades fundamentales - Toomates · Segunda parte: Desigualdades geométricas. 10 Desigualdades con los lados del triángulo. → Desigualdad Triangular. La Transformación
Page 35: 1 Las desigualdades fundamentales - Toomates · Segunda parte: Desigualdades geométricas. 10 Desigualdades con los lados del triángulo. → Desigualdad Triangular. La Transformación

9 Desigualdades trigonométricas.

9.1 F

Demuestra que

221cos

1sin

1 abx

b

x

a

para cualquier xba ,, con 0, ba y 2

0

x

9.2 F

Determina el valor mínimo de

xx

xxxf

sin

4sin9)(

22

para x0 AIME 1983 #9

9.3 MF

Determina el valor mínimo de

x

x

x

xxf

sin

cos

cos

sin)(

33

2

0

x

9.4 F

Demuestra que

221cos

1sin

1 abx

b

x

a

para todo xba ,, tales que 0, ba y 2

0

x .

Page 36: 1 Las desigualdades fundamentales - Toomates · Segunda parte: Desigualdades geométricas. 10 Desigualdades con los lados del triángulo. → Desigualdad Triangular. La Transformación
Page 37: 1 Las desigualdades fundamentales - Toomates · Segunda parte: Desigualdades geométricas. 10 Desigualdades con los lados del triángulo. → Desigualdad Triangular. La Transformación

Segunda parte: Desigualdades geométricas.

10 Desigualdades con los lados del triángulo.

Dedicaremos este primer tema del estudio de las desigualdades geométricas a la

Desigualdad triangular y a la Transformación de Ravi.

10.1 Teorema. La Desigualdad Triangular

Cuando trabajamos con triángulos debemos tener muy en cuenta que los números

0,, cba deben cumplir, además, la Desigualdad Triangular (ver GA/3.7.7):

La suma de dos lados siempre es mayor que el lado restante

a) cba , acb , bac

O equivalentemente:

b) cba , cab , bac

c) 0))()(( bacacbcba

Demostración:

ba )

babacbacbaacb

abccabacb

aacbacbacba

2

00

20

Las otras dos condiciones se demuestran de forma análoga.

ca )La expresión ))()(( bacacbcba es el producto de tres factores

positivos, por lo tanto es un número positivo.

ac )Por hipótesis, 0))()(( bacacbcba , y por tanto, o bien los tres

factores son positivos, es decir, se cumplen las condiciones del apartado a, o bien dos de

ellos son negativos y el restante positivo. Pero esto no puede suceder. Por ejemplo, si

0200

0

bbacbcba

acb

cba

Lo cual no puede suceder pues estamos trabajando con valores positivos.

Page 38: 1 Las desigualdades fundamentales - Toomates · Segunda parte: Desigualdades geométricas. 10 Desigualdades con los lados del triángulo. → Desigualdad Triangular. La Transformación

10.2 Teorema. La Transformación de Ravi.

Las condiciones sobre los lados 0,, cba que acabamos de ver pueden ser sustituidas

por la existencia de tres valores 0,, zyx tales que

yxa , zyb , xzc ,

Demostración. Sea 2/)( cbas . Basta tomar

02/)(2/)( bcabcbabsx

02/)(2/)( cbaccbacsy

02/)(2/)( acbacbaasz

Y está claro que, por ejemplo, acbcbacbsyx 2

cczzxyzyyxba 22

y las otras dos condiciones se demuestran de forma similar.

10.3 Observación.

La Transformación de Ravi equivale geométricamente a determinar los puntos de

tangencia entre el triángulo y su circunferencia inscrita. (ver GA/11.4.10)

10.4 Observación.

Es posible expresar ciertos valores notables de la geometría del triángulo en función de

las zyx ,, de la Transformación de Ravi:

a) zyxcba

s

2

b) Por la fórmula de Heron: xyzzyxcsbsassABC )())()((

c) Puesto que zyx

xyz

zyx

xyzzyx

s

ABCrrsABC

)(

d) xyzzyx

xzzyyxR

R

abcABC

4

))()((

4

Otras identidades que pueden ser útiles:

e) 22 )(4)()(2 zyxcbazyxcba

f) )(3222 xzyzxyzyxcabcab

Page 39: 1 Las desigualdades fundamentales - Toomates · Segunda parte: Desigualdades geométricas. 10 Desigualdades con los lados del triángulo. → Desigualdad Triangular. La Transformación

En los siguientes problemas se pide demostrar la desigualdad indicada, y siempre se supone

que cba ,, son los lados de un triángulo.

10.5

22222 cbacba

10.6

acbcabcbaacbcab 2222

10.7

a) 3

cba

c

bac

b

acb

a b) 3

c

cba

b

bac

a

acb

10.8

cbabacacbcba

y determinar cuándo ocurre la igualdad.

Asian Pacific Mathematics Olympiad 1996, Problema #5

10.9 M

Demuestra que, en todo triángulo ABC , se cumple rR 2 , donde R es el circunradio

y r el inradio del triángulo.

10.10 D

abccbacbacbacba 3)()()( 222

IMO 1964 #2

10.11 M

bcacba 2

10.12 F

cbacbabacacb

10.13 F

0222 acaccbcbbaba

IMO 1983 #6

10.14 MF

)(4)()(3 2 cabcabcbacabcab

Page 40: 1 Las desigualdades fundamentales - Toomates · Segunda parte: Desigualdades geométricas. 10 Desigualdades con los lados del triángulo. → Desigualdad Triangular. La Transformación

10.15 MF

)(2222 cabcabcbacabcab

10.16 D

22

3

ba

c

ac

b

cb

a

10.17 D

abccbacbacbacba 3222222222

10.18 M

8

1

ac

ac

cb

cb

ba

ba

10.19 MF

Si, además, se satisface 3 cabcab , demostrar que

323 cba

10.20 F

Si s es el semiperímetro del triángulo,

a) abbsas ))((

b) 4

))(())(())((cabcab

ascscsbsbsas

Page 41: 1 Las desigualdades fundamentales - Toomates · Segunda parte: Desigualdades geométricas. 10 Desigualdades con los lados del triángulo. → Desigualdad Triangular. La Transformación

11 Desigualdades geométricas con trigonometría.

11.1 M

Dado un triángulo ABC , demuestra que:

a) 12

tan2

tan2

tan2

tan2

tan2

tan ACCBBA

b) 9

3

2tan

2tan

2tan

CBA

11.2 MF

Demuestra que en cualquier triángulo ABC

bc

aA

22sin

11.3 F

Dos circunferencias, 1 y 2 , tienen radio 5 y 12 respectivamente, y la distancia entre

sus centros es de 13 unidades. Las circunferencias se cortan en los puntos P y Q. Se

traza una recta l que pasa por P y corta la circunferencia 1 en PX y 2 en PY .

Determina el máximo valor de PYPX .

West Windsor Plainsboro Math Tournament 2013

11.4 F

Sea ABC un triángulo acutángulo. Demostrar que:

a) CBACBA tantantantantantan .

b) 33tantantan CBA .

Page 42: 1 Las desigualdades fundamentales - Toomates · Segunda parte: Desigualdades geométricas. 10 Desigualdades con los lados del triángulo. → Desigualdad Triangular. La Transformación

11.5 M

Dado un triángulo ABC , demostrar que:

a) cb

aA

2sin

b) 8

1

2sin

2sin

2sin

CBA

c) 12

sin2

sin2

sin22

sin2

sin2

sin 222 CBACBA

d) 4

3

2sin

2sin

2sin 222

CBA

e) 4

9

2cos

2cos

2cos 222

CBA

f) 4

33

2cos

2cos

2cos

CBA

g) 62

csc2

csc2

csc CBA

11.6

Supongamos que en un triángulo ABC se cumple

1sinsinsin CBA

Demuestra que º30,,min ACCBBA

Page 43: 1 Las desigualdades fundamentales - Toomates · Segunda parte: Desigualdades geométricas. 10 Desigualdades con los lados del triángulo. → Desigualdad Triangular. La Transformación

12 Desigualdades con las rectas del triángulo.

12.1 Comparación de ángulos y lados (GA/3.7.7)

Lado mayor determina ángulo contrario mayor y viceversa.

12.2 La desigualdad de Ptolomeo. (GA/13.2.7)

Dados cuatro puntos DCBA ,,, se cumple

BDACDABCCDAB

y la igualdad solo sucede si DCBA ,,, están alineados o son cocíclicos en este orden.

12.3 Teorema y desigualdad de Euler. (GA/11.13.1)

Dado un triángulo ABC , sea O el circuncentro, I el incentro, r el inradio y R el

circunradio. Se cumple:

)2(2 rRROI , y por tanto rR 2

y la igualdad acontece si y solo si el triángulo es equilátero.

12.4 Teorema y desigualdad de Leibniz. (GA/11.13.4)

En un triángulo ABC de lados cba ,, , con circuncentro O, baricentro G y circunradio

R, se cumple:

22222

9

1cbaROG , y por tanto 22229 cbaR

y la igualdad acontece si y solo si GO , es decir, cuando el triángulo es equilátero.

12.5 F

Dado un triángulo ABC con la mediana AD , se cumple:

a) 2

acbAD

b) Si aAD2

1 , entonces º90BAC .

12.6 F

Siendo a, b y c los lados de un triángulo y T su área, demostrar que

Tcba 34222

¿Cuándo se da la igualdad? IMO 1961 #2

12.7 D

Demostrar que en un triángulo ABC de lados cba ,, se cumple

cba

abcABC

934

Page 44: 1 Las desigualdades fundamentales - Toomates · Segunda parte: Desigualdades geométricas. 10 Desigualdades con los lados del triángulo. → Desigualdad Triangular. La Transformación

12.8 D

Sea P un punto en el interior de un triángulo ABC con circunradio R. Sean rqp ,, las

distancias de P a los lados BC, AC y AB, respectivamente. Demostrar que se cumple

R

cbarqp

2

222

12.9 F

Sea ABC un triángulo equilátero de lado a , sea P un punto del su interior y sean D, E

y F las proyecciones de M en los lados BC, CA y AB, respectivamente. Demostrar que:

a) hPFPEPD , donde h es la altura del triángulo ABC . ("Lema de Viviani").

b) aPFPEPD

36111

c) aPDPFPFPEPEPD

33111

12.10 F

Sean ah , bh y ch las longitudes de las correspondientes alturas por A, B y C de un

triángulo ABC dado, y sea r el radio de su incírculo. Demostrar que:

a) 1cba h

r

h

r

h

r b) rhhh cba 9

12.11 F

Sea ABC un triángulo con alturas AD, BE y CF, y sea H su ortocentro. Demostrar

que:

a) 9HF

CF

HE

BE

HD

AD b)

2

3

HC

HF

HB

HE

HA

HD

Page 45: 1 Las desigualdades fundamentales - Toomates · Segunda parte: Desigualdades geométricas. 10 Desigualdades con los lados del triángulo. → Desigualdad Triangular. La Transformación

12.12 MD

Las longitudes de los lados de un hexágono ABCDEF satisfacen BCAB , DECD

y FAEF . Demuestra que

2

3

FC

FA

DA

DE

BE

BC

IMO 1997 Shortlist #7

12.13 MD

Sea ABC un triángulo y sean L, M y N puntos en BC, CA y AB, respectivametne.

Sean P, Q y R los respectivos puntos de intersección de las rectas AL, BM y CN con el

circuncírculo de ABC . Demostrar que

9NR

CN

MQ

BM

LP

AL

COREA 1995

12.14 MF

Sean AD, BE y CF las alturas de un triángulo ABC , y sean PQ, PR y PS las distancias

entre un punto P a los lados BC, CA, AB, respectivamente. Demostrar que

9PS

CF

PR

BE

PQ

AD

Page 46: 1 Las desigualdades fundamentales - Toomates · Segunda parte: Desigualdades geométricas. 10 Desigualdades con los lados del triángulo. → Desigualdad Triangular. La Transformación

12.15 M

Las cevianas AL, BM y CN de un triángulo ABC se cortan en un punto P. Demostrar

que

6PN

CP

PM

BP

PL

AP

si y solo si el punto P es el baricentro del triángulo.

12.16 MF

Las alturas AD, BE y CF cortan el circuncírculo de un triángulo ABC en los puntos

D', E' y F', respectivamente. Demostrar que:

a) 9'''

FF

CF

EE

BE

DD

AD b)

4

9

'''

CF

CF

BE

BE

AD

AD

12.17 D

Sean cba lll ,, las longitudes de las bisectrices internas de los ángulos de un triángulo, y

sean s y r el semiperímetro y el inradio del triángulo. Demostrar que:

a) 2srlll cba

b) 2222 slll cba

c) 2sllllll accbba

12.18 MF

Dado un triángulo ABC , sean M, N y P puntos arbitrarios en las rectas BC, CA y AB,

respectivamente. Si denotamos por cba ,, las longitudes de los lados del triángulo y por

R el circunradio, demostrar que

RCP

ab

BN

ca

AM

bc6

12.19 F

Demostrar que si R es el circunradio y denotamos por cba ,, las longitudes de los lados

del triángulo, se cumple:

2

1111

Racbcab

Page 47: 1 Las desigualdades fundamentales - Toomates · Segunda parte: Desigualdades geométricas. 10 Desigualdades con los lados del triángulo. → Desigualdad Triangular. La Transformación

13 Inecuaciones.

13.1

M

Resuelve la siguiente inecuación:

92

211

42

2

xx

x

IMO 1960 #2

13.2 D

Determina todos los números reales para los que se satisface la inecuación:

2

113 xx

IMO 1962 #2

13.3 F

Dados 1 ba , resuelve la inecuación

bx

ax

bx

ax

1

1

Page 48: 1 Las desigualdades fundamentales - Toomates · Segunda parte: Desigualdades geométricas. 10 Desigualdades con los lados del triángulo. → Desigualdad Triangular. La Transformación
Page 49: 1 Las desigualdades fundamentales - Toomates · Segunda parte: Desigualdades geométricas. 10 Desigualdades con los lados del triángulo. → Desigualdad Triangular. La Transformación

14 Aplicación de las desigualdades en la resolución de ecuaciones.

14.1 M

Encontrar todas las soluciones enteras positivas de

12

1111

cbaaccbba

OME 2017 Fase Local #4

14.2 D

Sean nba ,, enteros positivos tales que ba y 21 nab . Prueba que

34 nba

Indica justificadamente cuando se alcanza la igualdad.

OME Fase nacional 2013 #1

Page 50: 1 Las desigualdades fundamentales - Toomates · Segunda parte: Desigualdades geométricas. 10 Desigualdades con los lados del triángulo. → Desigualdad Triangular. La Transformación
Page 51: 1 Las desigualdades fundamentales - Toomates · Segunda parte: Desigualdades geométricas. 10 Desigualdades con los lados del triángulo. → Desigualdad Triangular. La Transformación

Soluciones.

1.2

Partimos de 1.1c : a

b

b

a

ab

babaab

2222 22

1.3

2sin

cos

cos

sin

cossin

cos

cossin

sin

cossin

cos

cossin

sin

cossin

cossin

cossin

1 222222

x

x

x

x

xx

x

xx

x

xx

x

xx

x

xx

xx

xx

1.4

Es un caso particular de 1.2 tomando 1b .

1.5

21

11

1

1

1

1

1

2

2

2

22

2

2

2

xx

xx

x

x

x por aplicación directa de 1.4

2.3

Aplicamos la desigualdad AM-GM tres veces:

abccbaacbcabcacbba

acca

bccb

abba

222

8

))()((

2

2

2

2.4

12

1...

2

1

2

1

12

1...1121...11

21

2121

n

n

nn

n

aaa

aaaaaa

Pero, aplicando la desigualdad AM-GM: iii aa

a

1

2

1

Luego 11......2

1...

2

1

2

12121

21

nnn aaaaaa

aaa

2.5

a) Primera versión: Aplicando la desigualdad AM-GM:

abbaba

22

22

2 , acca

ca

2222

2 , bccb

cb

2222

2

Luego

Page 52: 1 Las desigualdades fundamentales - Toomates · Segunda parte: Desigualdades geométricas. 10 Desigualdades con los lados del triángulo. → Desigualdad Triangular. La Transformación

bcacabcba

bcacabcba

bcacabcbcaba

222

222222222

2

222

222

Segunda versión:

0)()()(0222222

0

222222

222222

cbcabaacbcabcba

acbcabcbaacbcabcba

Reduciendo la desigualdad al modelo 1.1b

b) Basta aplicar el apartado anterior teniendo en cuenta que

)(22222acbcabcbacba , y por tanto:

)(2)(2)(2 222222222 cbacbaacbcabcbaacbcabacbcab

2.6

Utilizamos la siguiente factorización:

acbcabcbacba

acbcabcbacbaabccba

222

222333 ))((3

Y aplicando el ejercicio anterior:

abccba

abccbaacbcabcbacba

acbcabcbaacbcabcba

3

030

0

333

333222

222222

Ahora, mediante el siguiente cambio 333 ,, ccbbaa llegamos a la desigualdad

deseada.

2.8

Aplicamos la desigualdad AM-GM:

n

nn

nnn

n

aaaa

n

aaaa

......

...... 1

11

1

Aplicada a los números naaa n ,...,2,1 21 , y teniendo en cuenta que

2

)1(...21

nnn

nnn

n

n

nn

n

nnn

2

12/)1(...1...21!

2.9

Aplicamos la desigualdad AM-QM:

33

222 zyxzyx

222222

2222

34123

24333

62 zyx

zyxzyxzyx

Page 53: 1 Las desigualdades fundamentales - Toomates · Segunda parte: Desigualdades geométricas. 10 Desigualdades con los lados del triángulo. → Desigualdad Triangular. La Transformación

2.10

acbcabcba

acbcabacbcabcbacba

2

1)(

)(21)(2)(

2

2222

Aplicamos la desigualdad AM-QM:

333

3

3

1

33

2222

cbacbacbacba

Luego 12

13

2

1)( 2

cba

acbcab

Por otro lado: 0)( 2 cba , luego acbcabcba

2

1)(

2

10

2

1 2

2.11

22

2

2222 ba

baba

ba

ba

Aplicamos la desigualdad AM-QM:

22

2

22222222

2

242222

baba

babababababa

2.12

Sean acbx , bcay , y cbaz .

Entonces

22

yxccbcaacbyx

,

y de la misma manera: 2

zxb

,

2

zya

Aplicamos la desigualdad AM-GM:

azy

yzbzx

xzcyx

xy

2

,2

,2

Luego

abczyzxyx

yzxzxyxyxzyzzyx

xyzcbabcaacb

222

))()((

222

2.13

Aplicamos la desigualdad AM-GM:

Page 54: 1 Las desigualdades fundamentales - Toomates · Segunda parte: Desigualdades geométricas. 10 Desigualdades con los lados del triángulo. → Desigualdad Triangular. La Transformación

88

8

2

2

2

b

c

b

a

a

c

a

b

c

b

c

a

b

ca

a

cb

c

ba

b

c

b

a

a

c

a

b

c

b

c

a

b

ca

a

cb

c

ba

b

c

b

a

b

c

b

a

b

ca

a

c

a

b

a

c

a

b

a

cb

c

b

c

a

c

b

c

a

c

ba

2.14

Aplicamos la desigualdad AM-GM por separado en cada paréntesis:

abcdcbaaccbbaaccbba 3333 3333 222222

abcdcbacabcab 333 333222

Multiplicando estas dos desigualdades llegamos al resultado deseado.

2.15

Nos vamos a basar en la siguiente desigualdad: ac

b

b

a

b

a3

En efecto, por la desigualdad AM-GM:

aaabccba

cbacbacbac

b

b

a

b

a

c

b

b

a

b

a

333

13333

3 333 112

33 1123 1123 1123

y de la misma manera: ba

c

c

b

c

b3 y c

b

a

a

c

a

c3

Por tanto

cbaa

c

c

b

b

a

cbacbab

a

a

c

a

c

a

c

c

b

c

b

c

b

b

a

b

a

a

c

c

b

b

a

)(33333

2.16

Aplicamos la desigualdad AM-GM dos veces: BAABBA

AB

22

byaxxyabbyaxxyababxybyaxbyax

xyabxyab

224

2

2

2.17

Aplicando la desigualdad AM-GM tres veces:

Page 55: 1 Las desigualdades fundamentales - Toomates · Segunda parte: Desigualdades geométricas. 10 Desigualdades con los lados del triángulo. → Desigualdad Triangular. La Transformación

222

222

222222

222222

222

22222

22222

22222

2

2

2

222

2

2

2

babcaccbaabc

babcaccbaabc

acbccababcaccbaabc

acbccababcacbacbcaabc

acbcacbcbac

cabacababca

bcacbcacabc

2.18

Aplicamos la desigualdad AM-GM:

444

244442

44

22

2222

22

cossin

1cossincossincossin

xx

xxxxxx

Y la igualdad la encontramos cuando

4

1tan1tan

1cos

sin1

cos

sin1

cos

sincossin44

2

22

2

222cossin 22

xxx

x

x

x

x

x

xxxxx

2.19

Aplicando la desigualdad AM-GM,

abcbc

acb

bc

a333

33

Y de la misma manera, bcaac

b3

3

y cbaab

c3

3

.

Sumando las tres igualdades siguientes tenemos

Page 56: 1 Las desigualdades fundamentales - Toomates · Segunda parte: Desigualdades geométricas. 10 Desigualdades con los lados del triángulo. → Desigualdad Triangular. La Transformación

cbaab

c

ac

b

bc

acbacba

ab

c

ac

b

bc

a

cbacbabaab

cca

ac

bcb

bc

a

333333

333

32

3333

2.20

Aplicando la desigualdad AM-GM:

accbbacba

acaccacc

cbcbbcbb

babaabaa

222333

23 333333

23 333333

23 333333

33

33

33

33

2.21

Aplicando la desigualdad AM-GM:

babaabaaba 2242323 222

Y de la misma manera:

cbbcb 223 2

accac 223 2

Y solo nos queda sumar las tres desigualdades anteriores.

2.22

Aplicamos la desigualdad AM-GM:

3273

27

2

27

2

23

27

2

27

2

23

2

3

32

3

2

3 xxzyzy

zy

xzyzy

zy

x

Y por lo tanto:

27

429

27

229

27

2

27

2

322

3 zyxzyzyxzyzyx

zy

x

De la misma manera:

27

429

22

3 xzy

xz

y

, 27

429

22

3 yxz

yx

z

Sumando las tres desigualdades anteriores llegamos a

3

1

9

3

927

333

27

429429429

27

429

27

429

27

429

2222

3

2

3

2

3

zyxzyxyxzxzyzyx

yxzxzyzyx

yx

z

xz

y

zy

x

Fuente de la solución: "2010 IMO Summer IMO Training: Inequalities Adrian Tang". pág. 2

2.23

Aplicando la desigualdad GM-AM,

Page 57: 1 Las desigualdades fundamentales - Toomates · Segunda parte: Desigualdades geométricas. 10 Desigualdades con los lados del triángulo. → Desigualdad Triangular. La Transformación

a

bcbc

acb

aa

cb2

21

Y aplicando el mismo principio a los otros dos sumandos llegamos a

(*)

2

a

bc

c

ab

c

ab

ab

ac

ab

ac

a

bc

c

ab

ab

ac

a

bc

c

ba

b

ac

a

cb

De nuevo, aplicando la desigualdad GM-AM:

cab

ac

a

bc

ab

ac

a

bc22

2/1

Y aplicando el mismo principio a los otros dos sumandos llegamos a

cbacbacba 2(*)

Finalmente, basta tener en cuenta que, aplicando la desigualdad GM-AM:

3332/12/1

abccbacba

2.24

222

2222

92

29

cbaacbcab

acbcabcbacba

92

922

222

222

cbacba

cbacabcabcbacabcabcba

Lo cual es cierto, pues aplicando la desigualdad AM-GM:

aaaaaaaaa 3)(32 3/1222

Y de la misma forma con las otras dos variables llegamos a

933)(33332 222 cbacbacbacba

3.3

Basta aplicar la Desigualdad GM-HM:

)(22

2

2222

22

yxyx

yx

yxyxbaba

yb

xa

3.4

Page 58: 1 Las desigualdades fundamentales - Toomates · Segunda parte: Desigualdades geométricas. 10 Desigualdades con los lados del triángulo. → Desigualdad Triangular. La Transformación

Utilizamos la desigualdad del problema 3.3. En nuestro caso

345782)722144(2722144 xxxx

Y la igualdad acontece cuando 433722144 xxx

El dominio de definición de esta función es 722,144722144 xx

4.5

Tomamos 1...21 naaa ,

n

bbb

n

bbb

n

bbb

n

bbbn

n

bbb

n

bbbnbbbbbbn

bbbbbbn

bbbbbb

nnnn

nn

nn

nn

nn

............

............

......

......1...11

21

22

2

2

121

2

22

2

2

1

21

22

2

2

12

21

22

2

2

1

2

21

22

2

2

1

2

21

22

2

2

1

4.6

Tomando zyxaaa ,,,, 321 , y zyx

bbb3

,2

,1

,, 321

y aplicando la desigualdad Cauchy-Schwarz, tenemos:

36321941941 2

zyxzyx

zyx

Y la igualdad solo ocurre cuando 321

zyx , o

2

1,

3

1,

6

1,, zyx

4.7

2222222222 )(cossincossin)( baxfbaxxbaxbxaxf

y este máximo aparece para baxxx

x

b

a/arctantan

cos

sin

4.8

Por la Desigualdad Cauchy-Schwarz:

5/1600)516(05164641664

1648

1111

222

22

222222222

eeeeeeee

ee

dcbadcba

5/16

00)516( eee

Page 59: 1 Las desigualdades fundamentales - Toomates · Segunda parte: Desigualdades geométricas. 10 Desigualdades con los lados del triángulo. → Desigualdad Triangular. La Transformación

El valor máximo es 5

16e , que ocurre cuando dcbakdcba )11,1,1(),,,( , y por

tanto 5

68

5

164 dcbaa

4.9

222

22222222

3

31111113

cba

cbacbacba

4.10

Tomando accbbau ,, y abcabcv ,,

accbbaaccbba 222222

abcabcabcabc 222222

222222

93 cbaabcabcabcabcabaccacbbcba

Y es, por tanto, una aplicación directa de la desigualdad Cauchy-Schwarz.

Observación. Este problema se podría haber resuelto aplicando la desigualdad GM-HM:

Dividiendo los dos lados por 222 cba , la desigualdad se transforma en

9

b

a

a

c

c

b

b

c

a

b

c

a

Y aplicando la Desigualdad GM-HM: b

c

a

b

c

ab

c

a

b

c

a

b

c

a

b

c

a

33

11 33

Y de la misma forma b

a

a

c

c

b3

4.11

abcdabcabdacdbcddcba

dcbaabcd

abcabdacdbcd

dcba

64164

6416416411

Tomando las cuaternas ),,,( dcbau y )16,4,,( abcabdacdbcdv

abcdabcdabcdabcdabcdabcdvu 64842222

y vemos que es una aplicación directa de la desigualdad Cauchy-Schwarz.

4.12

Aplicamos la Desigualdad Cauchy-Schwarz "en la forma de Engel" 4.3:

Page 60: 1 Las desigualdades fundamentales - Toomates · Segunda parte: Desigualdades geométricas. 10 Desigualdades con los lados del triángulo. → Desigualdad Triangular. La Transformación

cabcabcbacba

cbacba

cbacbac

c

b

b

a

a

23

3

111111

2222

2222

2222

2

2

2

2

2

2

4.13

Basta elevar al cuadrado y desarrollar algebraicamente:

22121

2

2

2

2

2

1

2

1

2121

2

2

2

2

2

1

2

1

21

2

2

2

121

2

2

2

1

2

2

2

2

2

1

2

1

2

2

2

2

2

1

2

1

2

21

2

21

2

2

2

2

2

1

2

1

2

2

2

2

2

1

2

1

2

21

2

21

22

2

2

2

2

1

2

1

2

21

2

21

2

2

2

2

2

1

2

1

22

222

2

bbaababa

bbaababa

bbbbaaaababababa

bbaababababa

bbaababa

bbaababa

Que es la desigualdad Cauchy-Schwarz.

4.14

Observamos que

222

2

222

2222

222

22

222

2

2

21

2

222

2

2

2

)(

cba

cab

cba

cabcba

cba

abba

cba

ba

Luego

222

2

222

2

222

2

222

2

222

2

222

2

22223

2

)(

2

)(

2

)(

bac

bca

acb

abc

cba

cab

bac

ac

acb

cb

cba

ba

Luego nuestro problema se reduce a demostrar que

32

)(

2

)(

2

)(222

2

222

2

222

2

bac

ac

acb

cb

cba

ba

Aplicamos la desigualdad Cauchy-Schwarz "en la forma de Engel" (4.3):

22

2

22

2

222

2

2

)(

cb

b

ca

a

cba

ba

Y haciendo lo mismo en los otros dos sumandos llegamos a:

3

2

)(

2

)(

2

)(

22

22

22

22

22

22

22

2

22

2

22

2

22

2

22

2

22

2

222

2

222

2

222

2

ab

ab

cb

cb

ac

ca

ba

a

bc

c

ac

c

ab

b

cb

b

ca

a

bac

ac

acb

cb

cba

ba

Fuente de la solución: "Secrets in Inequalities (volume 1)" (Pham Kim Hung) pág. 35

Page 61: 1 Las desigualdades fundamentales - Toomates · Segunda parte: Desigualdades geométricas. 10 Desigualdades con los lados del triángulo. → Desigualdad Triangular. La Transformación

4.15

2

222

111111

111

11111

11

11

11121

zyxzyxz

z

y

y

x

x

zyxz

z

y

y

x

xzyx

z

z

y

y

x

x

zyxzyx

En donde hemos aplicado la desigualdad Cauchy-Schwarz "en la forma de Engel" (4.3).

4.16

2312

)2(3)1(26322

zyx

zyxzyx

Aplicamos la desigualdad Cauchy-Schwarz:

62312

666213212312 22

zyx

zyxzyx

Y la igualdad acontece cuando las ternas son proporcionales:

23

12

3

2

2

1

1 zx

yxzyx

1

11336323123

z

yxxxxxzyx

Es decir, cuando 1 zyx .

Observación: En las soluciones oficiales (SE pág. 1042) se presenta un razonamiento

alternativo mediante la desigualdad de Jensen aplicada a la función xxf )( .

5.1

0))((0

0

yxba

yxyx

baba

aybxbyaxaybxbyaxbybxayaxyxba ))((0

5.2

Puesto que la desigualdad es simétrica en las dos variables, podemos suponer yx .

xyxyyx

11110

xy

yx

xy

yx

xy

yx

x

y

y

x 111111 222222

Page 62: 1 Las desigualdades fundamentales - Toomates · Segunda parte: Desigualdades geométricas. 10 Desigualdades con los lados del triángulo. → Desigualdad Triangular. La Transformación

Y, aplicando el principio de Reordenación,

yxy

y

x

x

yy

xx

yy

xx

xy

yx

2222 111111

5.3

Desarrollando la expresión algébrica tenemos

bdbcadac

acabcdbdcdbcadabbdbcadac

cbaddbcadcba

2

))(())(())((0

Primera versión.

bcadbdac

bcadbdacbdbcadacbdbcadac

020

Si ba y dc , esta última desigualdad es una aplicación directa del Principio de

Reordenación.

Si ba y dc entonces cbdacbcada , contradiciendo la hipótesis del

enunciado cbda .

De la misma manera, si ba y dc , entonces

cbdacbdbdaba , contradiciendo de nuevo la hipótesis

cbda .

Finalmente, el caso ba y dc implica, por el principio de la Reordenación,

abbcacbd , y por lo tanto se satisface la desigualdad propuesta.

Segunda versión.

Basta tener en cuenta que ))(( dcbabdbcadac

y que dcbacbda

Luego 0)( 2 babdbcadac

5.4

Primera versión.

Sean 0ix y nnxxc ...1

Sean c

xa 1

1 , 2

212

c

xxa ,

3

3213

c

xxxa , ... , 1

...1 n

nn

c

xxa

y 1

1

1

ab ,

2

2

1

ab , ... , 1

1

n

na

b

Las secuencias ia y ib están ordenadas de forma opuesta. Luego:

12312111 ...... nnnnn babababababa

c

xxxx

c

x

c

x

c

x

c

x

xx

c

c

xxx

x

c

c

xx

c

x nn

.........1...1 321321

21

2

3

321

1

2

211

n

xxxxxxxxxxnc nn

nn

......... 321

1321

Segunda versión.

Page 63: 1 Las desigualdades fundamentales - Toomates · Segunda parte: Desigualdades geométricas. 10 Desigualdades con los lados del triángulo. → Desigualdad Triangular. La Transformación

Sin pérdida de generalidad podemos suponer que 1...21 naaa (ver Normalización, Tema 7).

Realizamos el cambio de variable 2

11

x

xa ,

3

22

x

xa , ...,

n

nn

x

xa 1

1

, con 0...,,, 21 nxxx ,

Luego 1x

xa n

n y el problema se reduce a demostrar nx

x

x

x

x

x

x

x n

n

n

1

1

3

2

2

1 ...

Observamos que la secuencia ),...,,( 21 nxxx es creciente, mientras que la secuencia

nxxx

1,...,

1,

1

21

es decreciente. Por lo tanto, aplicando el Principio de Reordenación,

nx

xx

xx

xx

xx

xx

xx

xx

x

x

x

x

x

x

x

n

nn

n

nn

n

n 1

...1111

...11

...2

2

1

1

1

1

3

2

2

1

1

1

3

2

2

1

5.5

Las secuencias ),,( cba y ),,( cba están ordenadas de la misma manera. Luego

acbcabacb

cba

cba

cbacba

222

y la igualdad se produce si y solo si cba

5.6

222

222222

333 cbbaacbac

cba

cba

cbacba

5.7

Sean cba ,, números reales positivos.

Las secuencias cba ,, y

baaccb

1,

1,

1 están ordenadas de la misma manera. Luego

cbbaac

cba

baaccb

cba111111

accbba

cba

baaccb

cba111111

Y sumando estas dos desigualdades llegamos a

Page 64: 1 Las desigualdades fundamentales - Toomates · Segunda parte: Desigualdades geométricas. 10 Desigualdades con los lados del triángulo. → Desigualdad Triangular. La Transformación

2

3

3))()(())()((

2

1111111112

ba

c

ac

b

cb

a

accbba

accbcaacbacbaccbba

ac

ca

cb

cb

ba

ba

ac

c

cb

b

ba

a

cb

c

ba

b

ac

a

ba

c

ac

b

cb

a

accbba

cba

cbbaac

cba

baaccb

cba

que es la desigualdad de Nesbitt (3.6).

5.8

Las secuencias ia y

ias

1 están ordenadas de la misma manera:

mn

mnmnasas

asasaa

11

Luego:

nk

asasas

aaa

asas

aa

kkk

n

n

n

...,,3,21...

11

...

1...

1

...

11

21

1

1

Luego, sumando las 1n desigualdades anteriores, llegamos (!!!!) al resultado deseado.

5.9

Primera versión.

bca

cbba

ca

cabcbba

ca

bac

baccabbcbacabcbba

))(()())((

)())((

2

22

Y de la misma forma

acb

caba

cb

abc

))((2

y cba

cacb

ba

cab

))((2

Luego

Page 65: 1 Las desigualdades fundamentales - Toomates · Segunda parte: Desigualdades geométricas. 10 Desigualdades con los lados del triángulo. → Desigualdad Triangular. La Transformación

)(2))(())(())((

))(())(())((

222

cbaba

cacb

cb

caba

ca

cbba

cbacba

cacba

cb

cabab

ca

cbba

ba

abc

ca

acb

cb

bca

Podemos suponer, sin pérdida de generalidad, que cba .

Y por tanto:

bacacbcbcaba

111

Y también ))(())(())(( cacbcbbacaba .

Luego, aplicando el criterio de Reordenación,

)(21

))((1

))((1

))((

1))((

1))((

1))((

cbacb

cacbba

cbbaca

caba

bacacb

cacbba

cbcaba

tal y como queríamos ver.

Segunda versión.

Podemos suponer, sin pérdida de generalidad, que cba .

Luego: 222 cba , y también bacacb

cbcaba

111

.

Luego, aplicando el criterio de Reordenación,

ac

c

cb

b

ba

a

ba

c

ac

b

cb

a

222222

Y por tanto:

cbaac

acc

cb

bcb

ba

aba

ba

ab

ca

ac

cb

bc

ac

c

cb

b

ba

a

ba

ab

ca

ac

cb

bc

ba

c

ca

b

cb

a

ba

abc

ca

acb

cb

bca

222

222

222

222

Fuente de esta versión: Secrets in Inequalities (volume 1) (Pham Kim Hung), pág. 92.

5.11

Primera versión.

Supongamos la secuencia ordenada dcba .

Entonces 2222 dcba ,

Page 66: 1 Las desigualdades fundamentales - Toomates · Segunda parte: Desigualdades geométricas. 10 Desigualdades con los lados del triángulo. → Desigualdad Triangular. La Transformación

Y también cbabadadcdcb ,

Y finalmente se cumple cba

d

bad

c

adc

b

dcb

a

2222

Entonces, aplicando la versión "inversa" de la desigualdad de Chebyshev:

cba

d

bad

c

adc

b

dcb

adcba

22222222 exp

4

14

Con )(3exp dcbacbabadadcdcb

Así pues,

cba

d

bad

c

adc

b

dcb

adcba

cba

d

bad

c

adc

b

dcb

adcba

2222

2222

3

16

4

34

Ya solo queda demostrar que 4 dcba , lo cual es cierto por el Corolario a la

desigualdad Cauchy-Schwarz:

dcbadcba

dcbadcba

4416

44

22

2

2222

Segunda versión.

Supongamos la secuencia ordenada dcba .

Entonces 2222 dcba ,

Y también cbabadadcdcb

Luego cbabadadcdcb

1111

Aplicamos la versión "directa" de la desigualdad de Chebyshev:

21

2222

expexp4

1

cba

d

bad

c

adc

b

dcb

a

Donde 4exp 2222

1 dcba

Y cbabadadcdcb

1111

exp 2

Ahora, se demuestra (!!!!) que

3

4

43

16

3

16

3

4exp

2222

2

dcbadcba

dcba

Fuente de la segunda versión: Secrets in Inequalities (volume 1) (Pham Kim Hung), pág. 54

6.5

Sea xxf )( . Esta función es cóncava, y por tanto podemos aplicar la Desigualdad de

Jensen:

Page 67: 1 Las desigualdades fundamentales - Toomates · Segunda parte: Desigualdades geométricas. 10 Desigualdades con los lados del triángulo. → Desigualdad Triangular. La Transformación

accbbaafccfbbfaacbcabfacbcab )()()(

Y ahora aplicamos

3

111)(3 22 acbcabcbaacbcab

Y aplicamos ahora que la función xxf )( es estrictamente creciente:

3

1

3

1

3

1 acbcabacbcab

Así pues, accbbaacbcab 3

1

6.6

Sean naaa ...,,, 21 números reales positivos.

Si uno de los valores ia es cero, la desigualdad AM-GM se cumple trivialmente. Luego

podemos suponer que 0...,,, 21 naaa .

Consideremos la función )log()( xxf . Es una función estrictamente creciente. Luego

n

aaaaaa

naaa

n

aaa

aaan

aaa

nn

nn

n

nn

n

log...loglog...log

1...log

...log

......

212121

21

2121

Es decir, queremos demostrar que

n

afafaf

n

aaaf nn )(...)()(... 2121

Que es precisamente la desigualdad de Jensen aplicada a la función cóncava )log()( xxf .

6.7

Observamos que la función

101

)(

xxxf es convexa en ,0 , pues su segunda derivada

es positiva. Aplicamos la desigualdad de Jensen:

9

101010

101010

3

10

3

103

3

133

3

13

33

3

)()()(3)()()(

111

fcba

f

cfbfafcfbfaf

cc

bb

aa

Este mínimo se alcanza cuando 3

1 cba . En efecto:

9

101010101010

3

10

3

1033

3

13

111

cc

bb

aa

Page 68: 1 Las desigualdades fundamentales - Toomates · Segunda parte: Desigualdades geométricas. 10 Desigualdades con los lados del triángulo. → Desigualdad Triangular. La Transformación

6.8

Sabemos que en todo triángulo ABC se cumple CBA , luego

33

1

3

1

3

1 CBA

Luego 13

1

3

1

3

1321 ttt y CBAxxx ,,,, 321 cumplen las condiciones de la

desigualdad de Jensen.

Por otro lado, xxf sin)( es una función cóncava en º1800 x , luego:

CBACBACBA sinsinsin3

1sin

3

1sin

3

1sin

3

1

3

1

3

1

3

1sin

3sin

2

3

O equivalentemente:

2

33sinsinsin CBA

6.9

Siguiendo con los elementos introducidos anteriormente,

xxf tan)( es una función convexa en º900 x , luego:

CBACBACBA tantantan3

1tan

3

1tan

3

1tan

3

1

3

1

3

1

3

1tan

3tan3

O equivalentemente:

CBA tantantan33

6.10

22

,2

,2

0,,0

CBA

CBA , y por tanto

2tan

2tan

2tan

666tan3

6tan3)30tan(3

3

133

CBACBACBA

Es decir:

2tan

2tan

2tan3

CBA

Con la igualdad si y solo si º60 CBA .

6.11

(*)...

...

...

...

)1)...(1)(1(

...

...

)1)...(1)(1(

...

...

21

21

21

21

21

21

21

21

21

21

n

n

n

n

n

n

n

n

n

n

n

n

n

n

n

aaan

aaa

aaan

aaa

aaa

aaa

aaan

aaa

aaa

aaa

Page 69: 1 Las desigualdades fundamentales - Toomates · Segunda parte: Desigualdades geométricas. 10 Desigualdades con los lados del triángulo. → Desigualdad Triangular. La Transformación

La función )ln()( xxf es creciente, luego

n

aaa

n

aaanaa

naaan

naaan

aaan

aaan

a

a

aaan

aaa

aaa

aaa

nnn

i

ii

n

n

n

nn

i i

i

n

n

n

n

n

...1ln

...ln1lnln

/)...(

/)...(ln

...

...ln

1ln

...

...ln

)1)...(1)(1(

...ln(*)

2121

1

21

21

21

21

1

21

21

21

21

Tomando la función xxxf 1lnln)( , la desigualdad anterior equivale a demostrar

n

aaafnaf n

n

i

i

...)( 21

1

Y, aplicando la desigualdad de Jersen, se reduce a demostrar que )(xf es convexa en

1,2/1 :

2

11221021)1(

1

)1(

1

0)1(

11)1()1)(1()1()(''

)1()1()1()('1lnln)(

222

22

22

22

1111

xxxxxxxxx

xxxxxf

xxxxxfxxxf

Fuente de esta solución: Secrets in Inequalities (volume 1) (Pham Kim Hung) pág. 70.

7.7

Vemos que la desigualdad es homogénea de grado 2 en todos sus términos excepto en la

constante 1. Pero aprovechando la condición 1 cba , tenemos 2211 cba , y la

podemos transformar en la siguiente desigualdad equivalente:

cabcabcba

cabcabcba

cabcabcabcabcbacba

cabcabcbacba

222

222

222222

2222

22

42

4

Que es una desigualdad ya demostrada.

7.8

En primer lugar observamos que 3

3 11abc

cbacbaabcabc

.

Ahora realizamos la siguiente sustitución: y

xa ,

z

yb ,

x

zc

x

z

z

y

y

x

x

zy

z

yx

y

zxcba

a

c

c

b

b

a

222

Page 70: 1 Las desigualdades fundamentales - Toomates · Segunda parte: Desigualdades geométricas. 10 Desigualdades con los lados del triángulo. → Desigualdad Triangular. La Transformación

Esta última desigualdad se demuestra fácilmente mediante el método de reordenación:

z

y

y

x

x

z

zyz

yxy

xxz

xyz

zxy

yxz

x

zy

z

yx

y

zx

222222222

111111

7.9

Vemos que esta inecuación no es homogénea, y procedemos a homogeneizarla:

111 3/13/1 abcabc

2223/1222 cbaabccbacbacba

Ahora desarrollamos la parte de la izquierda:

3/43/13/13/13/43/13/13/13/43/1cbacbacbaabccba

Luego queremos demostrar: 2223/43/13/13/13/43/13/13/13/4 cbacbacbacba

Aplicamos la desigualdad AM-GM apropiada:

663

2

6

6

4

666

3

663

2

2223/13/13/4

3/13/13/46 2286 222222222222

222222222

cbacba

cbacbacbaaaacbaaaa

cbacbacba

De la misma manera:

3

2

66,

63

2

6

2223/43/13/1

2223/13/43/1 cba

cbacba

cba

Y ya solo falta sumar las tres desigualdades anteriores, puesto que:

222222

222222222

222222222

6

666

6

4

6666

4

6666

4

3

2

6663

2

6663

2

cbacba

cbacbacba

cbacbacba

7.11

Puesto que la desigualdad es homogénea, podemos suponer que 1abc , y por tanto:

11

1

1

1

1

1333333

accbba

Con el cambio de variable 3ax , 3by , 3cz obtenemos la desigualdad equivalente

11

1

1

1

1

1

xzzyyx

Page 71: 1 Las desigualdades fundamentales - Toomates · Segunda parte: Desigualdades geométricas. 10 Desigualdades con los lados del triángulo. → Desigualdad Triangular. La Transformación

Y finalmente, con el cambio de variable 1 yxA , 1 zyB , 1 xzC llegamos

finalmente a

1111

CBA

(*)2)(2))()((

02)(2))()((

01)3222())()((

01)()1)(1)(1(0

11111

zyxxzzyyx

zyxxzzyyx

zyxxzzyyx

CBACBAABACBCABC

ABCABACBCABC

ABACBC

CBA

Ahora utilizamos la siguiente identidad:

)2)(()(2))()(( zxyzxyzyxxyzzyxxzzyyx

Luego

3)2)(((*) zxyzxyzyx

Finalmente, aplicamos la desigualdad AM-GM: 13

3

xyzzyx

1231)(3

3/23 222

zxyzxyzxyzxyxyzzyxzxyzxy

y ya solo nos queda multiplicar las dos desigualdades anteriores.

7.12

Vemos que

1888

),,(222

abc

c

cab

b

bca

acbaf

es una función homogénea de grado 0, luego podemos añadir la restricción 1 cba .

En efecto, supongamos que 10 mcba . Entonces

m

c

m

b

m

a

m

cba

1

Y realizamos el cambio de variable maa /' , mbb /' , mcc /' , obteniendo una

desigualdad equivalente a la primera:

cbafm

c

m

b

m

afcbaf ,,,,',','

Nota: La solución de este problema sigue en 6.4.

7.14

Realizaremos la misma sustitución que en el problema 7.2: y

xa ,

z

yb ,

x

zc

Page 72: 1 Las desigualdades fundamentales - Toomates · Segunda parte: Desigualdades geométricas. 10 Desigualdades con los lados del triángulo. → Desigualdad Triangular. La Transformación

xyzyxzxzyzyx

yxzxzyzyx

x

yxz

z

xzy

y

zyx

x

y

x

z

z

x

z

y

y

z

y

x

1

1

1111

Y esta desigualdad es el problema 2.12.

8.1

a) Observamos que 3

2

2

1 ,

5

4

4

3 ,

7

6

6

5 ... luego elevando al cuadrado, tenemos

10

1

100

1

100

99...

8

7

6

5

4

3

2

1

100

1

100

99...

9

8

8

7

7

6

6

5

5

4

4

3

3

2

2

1

100

99...

9

8

8

7

7

6

6

5

5

4

4

3

3

2

2

1

100

99...

8

7

8

7

6

5

6

5

4

3

4

3

2

1

2

1

100

99...

8

7

6

5

4

3

2

12

b) Por inducción en n:

Si 1n : 2

1

4

1

113

1

2

1

cierto.

Si 2n ; 64798737

1

8

3

7

1

123

1

4

3

2

1

cierto.

Suponiendo que es cierto para n, es decir:

13

1

2

12...

8

7

6

5

4

3

2

1

nn

n

queremos demostrar que también lo es para n+1:

nnnnnnn

nnnnnnnnnn

nnnnnn

n

n

nn

n

n

n

042028124192812

)484(1343144)22(131)1(3)12(

)22(131)1(3)12(1)1(3

1

22

12

13

1

1)1(3

1

22

12

2

12...

8

7

6

5

4

3

2

1

2323

2222

Lo cual es obviamente cierto.

c) Aplicando el apartado anterior: 12

1

155

115114412151144122

8.2

Las secuencias dcba ,,, y ),,,( 2222 dcba están ordenadas de la misma manera, luego

podemos aplicar la desigualdad de :

Page 73: 1 Las desigualdades fundamentales - Toomates · Segunda parte: Desigualdades geométricas. 10 Desigualdades con los lados del triángulo. → Desigualdad Triangular. La Transformación

4444

333322222222 dcbaddccbbaadcbadcba

Podemos volver a aplicar la d. s con la secuencias dcba ,,, y dcba ,,, :

444416

22222dcbaddccbbaadcbadcbadcba

Uniendo estas dos desigualdades llegamos a:

444164

333322222dcbadcbadcbadcbadcba

Es decir:

3333

333333

164164dcba

dcbadcbadcba

Tal y como queríamos ver.

8.3

Aplicamos la desigualdad Cauchy-Schwarz:

636191919369

3133

9

333

9

1

9

1

9

133

9

1

9

1

9

1333

9

13

9

13

9

13191919

2

22

222

222

2

2

cba

cbacba

cba

cbacba

Nota: Un desarrollo más limpio sería comparar las ternas 19,19,19 cba y )1,1,1( y

aplicar la desigualdad Cauchy-Schwarz:

63)(93191919111191919 cbacbacba

8.4

Aplicando la desigualdad Cauchy-Schwarz:

222

2

222222222

22

111

111111111111

acbabc

cba

acbacbcacabacbcabc

cba

Page 74: 1 Las desigualdades fundamentales - Toomates · Segunda parte: Desigualdades geométricas. 10 Desigualdades con los lados del triángulo. → Desigualdad Triangular. La Transformación

8.5

222222

222222

2

22

222

22

222

22

22222

1

1

1

1

1

1

111

111111

baaccb

aabcacbbcabacab

abc

acbcac

abc

abbcbc

abc

ac

c

ba

ba

bc

b

ca

ca

ab

a

cb

cb

ac

c

bc

b

ab

a

Aplicamos la desigualdad AM-GM:

3 222

3 222

3222

3222222

3222222222222

11144

3

111

13

111

13

111

13

1

1

1

1

1

13

1

1

1

1

1

1

cbacba

cbacbacba

baaccbbaaccb

cbacba

cbacba

cbacbacba

baaccbbaaccb

11121112

1111114

111

13

111

13

1

1

1

1

1

13

1

1

1

1

1

1

33

233 222

3222

3222222

3222222222222

Y esta última desigualdad es el problema 2.4.

Nota: En las soluciones oficiales el enunciado se reduce hasta la desigualdad de Nessbit (3.6).

8.6

Primera versión.

Observamos que no es posible que los tres números sean mayores que 1, pues

41,, 222 abccbacba , contradiciendo la hipótesis del enunciado.

Supongamos que 1a . Entonces 01 a y por tanto

0)1()( abccbaabccabcab

Veamos ahora la desigualdad contraria.

Puesto que los tres números no pueden ser mayores que 1, podemos suponer, sin pérdida de

generalidad, b y c son ambos menores o ambos mayores que 1, es decir:

0)1)(1( cb

Interpretando la igualdad 4222 abccba como una ecuación de segundo grado en a ,

tenemos que

Page 75: 1 Las desigualdades fundamentales - Toomates · Segunda parte: Desigualdades geométricas. 10 Desigualdades con los lados del triángulo. → Desigualdad Triangular. La Transformación

2

)4)(4(

2

)4(4)(04)(`

22222222 cbbccbbcbc

acbabca

Y por tanto:

bcabcacbaabccabcab )1)(1( .

Todo se reduce a demostrar que 2bca

(*)2

)4)(4(

2

2)4)(4(

2

)4)(4( 222222 bccbbccbbcbc

cbbca

Y se demuestra aplicando la desigualdad Cauchy-Schwarz en la forma:

2

2

2

1

2

2

2

12211 bbaababa

A nuestro problema:

41644)4()4()4)(4( 22222222 ccbbbccbbccb

Y por tanto

22

4(*) , tal y como queríamos ver.

Segunda versión.

La desigualdad inferior se demuestra igual que en la primera versión. Veamos la desigualdad

superior:

Igual que en la primera versión, podemos suponer sin pérdida de generalidad que b y c son

ambos mayores o iguales que 1 o ambos menores o iguales que 1, es decir, podemos suponer

que

0)1)(1( cb

Es conocido que bccb 222 , y por tanto

bca

abcaa

abca

abcaabcbcaabccba

2

)2(22

)2(4

)2(24

2

22222

Y por tanto:

2)1)(1(2122 bcabcbcaabcacaababccabcab

Fuente de estas dos versiones: https://artofproblemsolving.com/wiki/index.php/2001_USAMO_Problems/Problem_3

Nota: En dicha página web podemos encontrar una tercera versión mediante sustituciones

trigonométricas.

8.7

Primera versión.

Page 76: 1 Las desigualdades fundamentales - Toomates · Segunda parte: Desigualdades geométricas. 10 Desigualdades con los lados del triángulo. → Desigualdad Triangular. La Transformación

Realizamos el siguiente cambio de variable:

caz

cby

bax

Luego

acbcabcbazyx

accacaz

bccbcby

abbabax

222222

2222

2222

2222

2

2)(

2)(

2)(

Y por tanto:

2222222222 24 zyxacbcabcbacbacba

Por otro lado,

cxzy

bzyx

ayzx

2

2

2

9)(4)(4)(4

12)(4

1)(4

1)(4

1

124)(4)(4)(

124))((4))((4))((

12444444

3111

2

2

2

2

2

2

2

2

2

2

2

2

2

22

2

22

2

22

222

222222

z

xy

y

zx

x

yz

z

xy

y

zx

x

yz

z

xyz

y

zxy

x

yzx

z

yzxxzy

y

xzyzyx

x

zyxyzx

ac

ca

cb

bc

ba

ab

ac

ca

cb

bc

ba

ab

Por otro lado, 2,,,,44 222222 zyxzyxzyx

22

222 44

4404022,xx

yzyzyzyzyz

y de la misma forma

22

2 4)(4

yy

zx

,

22

2 4)(4

zz

xy

Luego

9111

4444)(4)(4)(4

2222222

2

2

2

2

2

zyxzyxz

xy

y

zx

x

yz

Page 77: 1 Las desigualdades fundamentales - Toomates · Segunda parte: Desigualdades geométricas. 10 Desigualdades con los lados del triángulo. → Desigualdad Triangular. La Transformación

Puesto que4

91114

222

222 zyx

zyx , tal y como queríamos ver.

Esta última desigualdad es el problema 3.3b.

Segunda versión.

Se sigue el mismo desarrollo, hasta llegar a

124)(4)(4)(

2

22

2

22

2

22

z

xyz

y

zxy

x

yzx

Puesto que, por hipótesis, 2224 zyx , seguimos así:

363

12322

2222

)()()(

4)(4)(4)(

222222

2222222

2

2

2

2

2

2

2

2

2

2

2

2

2

2

2

2

2

2

22222

2

22222

2

22222

2

22

2

22

2

22

z

xy

y

xz

x

yz

z

xy

y

xz

x

yz

z

xy

y

xz

x

yz

z

xy

y

xz

x

yz

z

z

y

y

x

x

z

xyz

y

xzy

x

yzx

z

xyz

y

xzy

x

yzx

z

zyxxyz

y

zyxzxy

x

zyxyzx

z

xyz

y

zxy

x

yzx

Y esta última desigualdad es consecuencia directa de aplicar la desigualdad AM-GM:

3133 33

222222

z

xy

y

xz

x

yz

z

xy

y

xz

x

yz

Tercera versión.

La condición 42222 cbacba se puede reescribir como

2222 acbcabcba

Y por lo tanto:

22

2

2

222

2

))((1

))(()(222

ba

bcac

ba

bcacba

ba

acbcabcbaab

ba

ab

Y por tanto:

3

))(())(())((

6222222

3111

222

222

222

ac

abcb

cb

caba

ba

bcac

ac

ca

cb

bc

ba

ab

ac

ca

cb

bc

ba

ab

Page 78: 1 Las desigualdades fundamentales - Toomates · Segunda parte: Desigualdades geométricas. 10 Desigualdades con los lados del triángulo. → Desigualdad Triangular. La Transformación

Pero esta última desigualdad se demuestra aplicando la desigualdad AM-GM:

3

)()()(3

))(())(())((3

2

222

222

ba

babcac

ac

abcb

cb

caba

ba

bcac

Fuente de las versiones 2 y 3: https://artofproblemsolving.com/wiki/index.php?title=2011_USAMO_Problems/Problem_1&oldid=78098

8.8

Puesto que todos los términos son homogéneos, podemos suponer, sin pérdida de generalidad,

que 3 cba , y por lo tanto la desigualdad a demostrar se transforma en la siguiente:

832

3

32

3

32

322

2

22

2

22

2

cc

c

bb

b

aa

a

Observamos que

963

68

3

1

32

681

3

1

32

6832

3

1

963

96

32

3222

2

2

2

22

2

aa

a

aa

a

aa

aaa

aa

aa

aa

a

Y esto mismo podemos hacer en los otros dos términos de la suma, obteniendo

7963

68

963

68

963

68

8963

68

3

1

963

68

3

1

963

68

3

1

222

222

cc

c

bb

b

aa

a

cc

c

bb

b

aa

a

Ahora observamos que

6

68

963

6866)1(3963

2

22

a

aa

aaaa

Y realizando esto mismo con los otros términos de la suma, llegamos a

76

1838

6

18)(8686868

6

1

6

68

6

68

6

68

963

68

963

68

963

68222

cbacba

cba

cc

c

bb

b

aa

a

Fuente de la solución: https://artofproblemsolving.com/wiki/index.php?title=2003_USAMO_Problems/Problem_5

Nota: En esta página web se presentan tres soluciones alternativas más.

8.9

223222122

3

2

2

2

1

12

1...

3

1

2

1

1

1...

321 naaaa

n

aaaa

k

an

nn

k

k

Por un lado, tenemos el conjunto ordenado

2222

1...

3

1

2

1

1

1

n

Sea nkkk ,...,, 21 la permutación de n,...,3,2,1 tal que nkkkk aaaa ...

321

Entonces, por el Principio de Reordenación,

Page 79: 1 Las desigualdades fundamentales - Toomates · Segunda parte: Desigualdades geométricas. 10 Desigualdades con los lados del triángulo. → Desigualdad Triangular. La Transformación

22222232221

1...

3

1

2

1

1

11...

3

1

2

1

1

1321 n

aaaan

aaaankkkkn

Puesto que la expresión de la derecha es el reodenamiento mínimo.

Pero naaaaaankkkkkk ,...,3,2,1

32121, y por tanto

22222222

1...

3

13

2

12

1

11

1...

3

1

2

1

1

1321 n

nn

aaaankkkk

n

1...

3

1

2

1

1

1

tal y como queríamos ver.

8.10

Primera versión.

abab 00

Aplicando la desigualdad AM-GM:

baabababababbabab

babab

bababb

babaab

222

2222

De nuevo aplicando la desigualdad AM-GM:

bababba

baababaaba

ababa

ababa

ababa

abbaba

ab

2

222

22

2

22

Así pues, tenemos la desigualdad

baabababba 2)(

Que, después de unas transformaciones apropiadas, se convertirá en la desigualdad del

enunciado:

b

baba

a

ba

b

baba

a

ba

b

baba

a

babaabababba

4422

222)(

22

222

2

b

baab

ba

a

ba

b

baabba

a

ba

b

baabba

a

ba

828

82

2

842

422

2222

Segunda versión.

Page 80: 1 Las desigualdades fundamentales - Toomates · Segunda parte: Desigualdades geométricas. 10 Desigualdades con los lados del triángulo. → Desigualdad Triangular. La Transformación

Observamos que 12

2

a

baabaabab

Y de la misma forma b

ba

21

, luego

b

ba

a

ba

b

ba

a

ba

b

ba

a

ba

41

4

21

221

2

22

22

b

baabba

a

ba

b

baba

a

ba

b

bababa

a

baba

82

2

8

44

44

22

22

2

222

22

Tercera versión.

Observamos que abba

ba

abba

abba

abba

4)(2

)(

2

2

2

2

2

Y por tanto la desigualdad del enunciado es equivalente a

babbaa 424 , que es trivial puesto que bababa 0

Fuente de la segunda y tercera versión: 101 Problems in Algebra from the training of the USA IMO team (Adreescu, Feng, 2001), página 29.

8.11

Haremos una demostración por reducción al absurdo. Supongamos que no se cumple la

desigualdad. Entonces:

12)12(241344)()( 22222 nbannnnnabbaba

Y puesto que ba, son enteros, llegamos a nba 2 .

Pero, por otro lado, por la desigualdad AM-GM,

222

22

2 12

2

21 nnn

nbaabn

, absurdo.

Así pues, se tiene que cumplir la desigualdad del enunciado, tal y como queríamos ver.

Veamos ahora la igualdad:

12)12(241344)()( 22222 nbannnnnabbaba

3434 nnba es un cuadrado perfecto impar 2)12(34 un para cierto entero

no negativo u , y por tanto

Page 81: 1 Las desigualdades fundamentales - Toomates · Segunda parte: Desigualdades geométricas. 10 Desigualdades con los lados del triángulo. → Desigualdad Triangular. La Transformación

14444144)12(34 2222 uunuunuuun , y 12 uba .

11121212

1222

uuuuunbunbaba

uba

nba

221121212 22 uuuubuauba

Y por tanto:

1112232122 22223422 nuuuuuuuuuab

Luego la igualdad se cumple cuando:

222 uua , 12 ub , 12 uun , 12 uba para todo entero no negativo u .

Fuente de esta solución: Solución oficial (SE)

8.12

Utilizaremos la desigualdad )(2 yxyx (ver Problema 3.3) y la igualdad acontece

si y solo si yx .

bababaabba

abba

ab

222

2222

)(22

22

La igualdad acontece cuando

bababaabbabaabba

ab

00)(0222

2222222

Nota: En las soluciones oficiales (SE) se presentan cuatro soluciones más.

8.13

Sea cbaS .

(*)63

1

3

1

3

176

3

7

3

7

3

7

63

322

3

322

3

322

623

22

3

22

3

2

3

2

3

2

3

2

332

3

323

3

233

3

aSbScSS

aS

S

bS

S

cS

S

aS

aSaS

bS

bSbS

cS

cScS

aS

aS

bS

bS

cS

cS

aS

aS

bS

bS

cS

cS

cba

cba

cba

cba

cba

cba

Por otro lado,

aSbScSSSSSaSbScS 3338

189333

Y por lo tanto

Page 82: 1 Las desigualdades fundamentales - Toomates · Segunda parte: Desigualdades geométricas. 10 Desigualdades con los lados del triángulo. → Desigualdad Triangular. La Transformación

8

1563

8

76

3

1

3

1

3

1333

8

7(*) 2

aSbScSaSbScS

En donde hemos utilizado la variación de la desigualdad AM-HM vista en 3.5a:

2

1

1

1...

1... n

aaaa

n

n

Fuente de esta solución: Solución oficial (SE).

8.14

Primera versión.

Si 0b , entonces 1a , y por tanto

22222xxbyaxbyax , se cumple la igualdad. Lo mismo sucede cuando 0a .

Supongamos que 0, ba . Es una aplicación directa de la Desigualdad Cauchy-Schwarz "en la

forma de Engel" (4.3)

2

2222222222 byaxba

b

yb

a

xa

b

yb

a

xa

b

yb

a

xabyax

La igualdad acontece solo cuando las parejas

b

by

a

ax, y ba , son proporcionales, es

decir, cuando

yxb

by

a

ax

bb

by

aa

ax

Segunda versión.

Podemos escribir

222

222222

222222222

)()2(

)2(22)1()1(

2

yxabxyyxab

xyyxababxybayabxabxyybbxaa

abxyybxabyaxbyaxbyax

Que es claramente no negativa. La igualdad acontece cuando

0)( 2 yxab , es decir, cuando 0a o 0b o yx .

Tercera versión.

En las soluciones oficiales se presenta una tercera versión mediante la Desigualdad de Jensen

aplicada a 2)( xxf .

Fuentes de la segunda y tercera versión: Soluciones oficiales (SE pág. 1031)

8.15

a) es la desigualdad AM-GM.

b)

Page 83: 1 Las desigualdades fundamentales - Toomates · Segunda parte: Desigualdades geométricas. 10 Desigualdades con los lados del triángulo. → Desigualdad Triangular. La Transformación

0)(02

222422

222

22222

2222

yxxyyx

xyyxyxyxyxyxyx

c) Aplicando el apartado b) tenemos

222

2

22

11

2

11

2

1111

2

1

2

11

211

pqpq

qp

qp

qq

pp

qq

pp

Luego el problema se reduce a demostrar que

pqqp

pqpqpqpqpqpq

22

1

2

1

4

1414

15

11

2

Que es precisamente el apartado a).

8.16

El problema equivale a demostrar que

022),( 2322 xyxyxyxxyxyxP si 1,0 yx

Escribimos el polinomio como polinomio en la variable x:

yxyyxyxyxP )21()12(),( 223

Dividimos este polinomio por 1x mediante el algoritmo de Euclides:

-1 12 y 221 yy

y

1 -1 y2 21 y

-1 y2 21 y 21 yy

Así pues,

)1()1()(

)1()(

11)1(2

11)1(2

1)1(12),(

2

22

222

222

222

yyxxyx

yyxxyx

yyxxyyxx

yyxxyyxx

yyxyyxxyxP

Y vemos que, efectivamente, para 1,0 yx los tres sumandos son positivos.

Fuente de la solución: Solución oficial (SE pág. 1022)

8.17

Page 84: 1 Las desigualdades fundamentales - Toomates · Segunda parte: Desigualdades geométricas. 10 Desigualdades con los lados del triángulo. → Desigualdad Triangular. La Transformación

2

)(2)1)(1(2

22

)(2)1)(1()()1()1()1)(1( 2222

baba

baab

bababaabbababaab

21)1)(1(

22

2

2)1)(1(2

22

baba

baab

baba

baab

Con lo que nuestro problema se reduce a demostrar que

12

1)1)(1(2

1,0

baba

baabba

Aquí utilizamos que 310 xxx , y que 12

01,0

ba

ba

Y por tanto, y aplicando la desigualdad AM-GM, tenemos

22

33

3

1

23

1

223

babababa

baab

baab

21

2111

3

1

21)1)(1(

21)1)(1( 3

ba

baba

baba

baba

Y finalmente:

12

122

1)1)(1(2

babababa

baab

8.18

a) Realizamos el siguiente cambio de variable:

11

a

ax

x

xa ,

11

b

by

y

yb ,

11

c

cz

z

zc

La desigualdad a demostrar ahora es 1222 cba , sujeta a la condición

acbcabcba

acbcabcbaabccbaabc

c

c

b

b

a

axyz

1

1)()1)(1)(1(

1111

Page 85: 1 Las desigualdades fundamentales - Toomates · Segunda parte: Desigualdades geométricas. 10 Desigualdades con los lados del triángulo. → Desigualdad Triangular. La Transformación

22222

2222

2222

2222

)1(1)(2)(1

)(2)(2

)()(2)(2

)()()1(2

cbacbacbacba

cbacbacba

cbacbacba

cbacbacba

De donde se deduce claramente que

11)1(0 2222222 cbacbacba

tal y como queríamos ver.

b) Seguimos con el cambio de variable anterior. Queremos resolver la ecuación

1222 cba

Y también 2222 )1(1 cbacba

Es decir, 1)1(10 2222 cbacbacba

Luego tenemos que resolver el sistema

1

1222

cba

cba

acbcab

acbcab

acbcabcbacba

0

)(211

)(2)(1

2

2222

0)1()1()1()1(

))(1()(011

2222

abaabbabaababbaa

babaabbacabbaccba

Entendiendo esta última expresión como una ecuación cuadrática en b, su discriminate es

)13)(1(

13)1(41)1()1(4)1()1(14)1( 22

aa

aaaaaaaaaaa

Para obtener valores racionales, es suficiente tomar valores de a racionales para los cuales

)13)(1( aa sea el cuadrado de un racional, de forma que el discriminante sea racional y

por tanto lo sea b y finalmente c).

Sea m

ka para ciertos enteros k y m .

2

)3)((3131)13)(1(

m

mkkm

m

mk

m

km

m

k

m

kaa

Luego nuestro problema se reduce a encontrar enteros mk, para los cuales km y mk 3

sean enteros.

Page 86: 1 Las desigualdades fundamentales - Toomates · Segunda parte: Desigualdades geométricas. 10 Desigualdades con los lados del triángulo. → Desigualdad Triangular. La Transformación

Esto se obtiene tomando, por ejemplo, 12 kkm , para cualquier k entero, pues entonces 22 )1(12 kkkkm

222 )1(12133 kkkkkkmk

y ambos son cuadrados.

Así pues,

m

kkmmkmkb

m

k

m

kk

2

)1()(

2

/)1()/1(

1)1()1(

22

2

2

22

Tomando el valor de b con la suma:

m

m

m

m

m

mmb

kkkkm

m

kkm

m

kkm

m

kkmb 1

2

22

2

2

1212

2

)1(

2

1

2

)1()(

22

222

Y, finalmente

m

k

m

k

mmm

k

m

m

m

kbaccba

11111

1111

La condición 1,, cba descarta los casos 0k y 1k .

Aunque no es necesario, deshaciendo el cambio de variable llegamos a

2)1(

k

kx , 2kky ,

2

1

k

kz

Fuente de la solución: Soluciones oficiales (SE , pág 964)

9.1

Vemos que la desigualdad se adapta al modelo de la desigualdad Cauchy-Schwarz:

22222 mqnpqpmn

Tomando 1 pn , x

am

sin y

x

bq

cos , que están bien definidos pues

1cos,sin02

0 xxx

2

22

cossin

11

cossin1

cossin11

cos1

sin1

xxab

xx

ab

x

b

x

a

x

a

x

a

Page 87: 1 Las desigualdades fundamentales - Toomates · Segunda parte: Desigualdades geométricas. 10 Desigualdades con los lados del triángulo. → Desigualdad Triangular. La Transformación

Solo queda demostrar que 2cossin

1

xx

En efecto,

2sin

cos

cos

sin

cossin

cos

cossin

sin

cossin

cossin

cossin

1 2222

x

x

x

x

xx

x

xx

x

xx

xx

xx

Por la desigualdad del problema 1.2

9.2

xxxx

xxxx

xx

xx

xxxf

sin

4sin9

sin

4

sin

sin9

sin

4sin9)(

2222

Sean xxu sin9 , xx

vsin

4

Observamos que su producto es constante: 36sin

4sin9

xxxxvu

Aplicamos la desigualdad AM-GM:

126362

vuuvvu

Luego el valor mínimo de la función es 12.

La igualdad se obtiene si 9

4sin

sin

4sin9 22 xx

xxxxvu

La función xxxg 22 sin)( cumple 0)0( g y 9

4

22

2

g , luego el valor

9

4 se

alcanzará entre 0 y 2

.

9.3

Vemos que las secuencias xx 33 cos,sin y

xx sin

1,

cos

1 están ordenadas en el mismo orden:

21

21212

3

1

3

cos

1

cos

1coscossinsinsinsin

20

xxxxxxxxx

Page 88: 1 Las desigualdades fundamentales - Toomates · Segunda parte: Desigualdades geométricas. 10 Desigualdades con los lados del triángulo. → Desigualdad Triangular. La Transformación

Luego , por el Principio de la Reordenación (ver Tema 5):

1cossin

cos

1

sin

1cossin

sin

1

cos

1cossin

)( 22

3333

xx

xx

xx

xx

xxxf

9.4

Primera versión. Aplicando la desigualdad Cauchy-Schwarz.

Vemos que se trata de una aplicación directa de la desigualdad Cauchy-Schwarz tomando

111 ba y x

aa

sin2 y

x

bb

cos2

x

b

x

a

xx

ab

x

b

x

a

x

b

x

a

cos1

sin1

cossin11

cos1

sin1

cossin11

2

2

Solo queda demostrar que 2

1cossin xx , lo cual se puede demostrar como aplicación de la

desigualdad GM-QM:

2

1cossin

2

1

2

cossincossin

22

xxxx

xx

O también se podría haber demostrado directamente mediante la identidad del seno del ángulo

doble:

2

11

2

12sin

2

1cossin xxx

Luego

22

21cossin

121cossin

1

2cossin

2cossin2

1cossin

abxx

abab

xx

ab

abxx

abab

xx

abxx

Con lo que llegamos finalmente a la desigualdad del enunciado.

Page 89: 1 Las desigualdades fundamentales - Toomates · Segunda parte: Desigualdades geométricas. 10 Desigualdades con los lados del triángulo. → Desigualdad Triangular. La Transformación

Segunda versión. Aplicando la desigualdad AM-GM.

Desarrollamos los dos lados de la desigualdad:

(*)cossinsincos

222

cossinsincos12221

cos1

sin121

2

xx

ab

x

a

x

babab

xx

ab

x

a

x

babab

x

b

x

aab

Pero, aplicando la desigualdad AM-GM,

x

a

x

b

xx

ab

sincoscossin2

y puesto que 2

1cossin xx (ver la primera versión)

abab

xx

abxx 2

2/1cossin2

1cossin

Y por tanto x

a

x

bab

sincos22

Por otro lado, está claro que xx

ababxx

cossin2

2

1cossin , y sumando estas dos

desigualdades llegamos a demostrar la desigualdad (*) .

Fuente de esta segunda versión: 103 Trigonometry Problems From the Training of the USA IMO Team (Titu Andreescu, 2005) , pág. 93.

10.5

Puesto que son los lados de un triángulo , podemos escribir

zya , xzb , yxc , con 0,, zyx

Luego

xzyzxyzyxcba

xzyzxyzyxxyxzxzyzy

yxyxxzxzzyzy

yxxzzycba

222222

222222

222222

222222

42

2222222

222

)()()(

xzyzxyzyx

zyzxzyzyyxxzxyx

zyxzyxyxxzzycba

24

4

4222

222

222

2222

Luego

Page 90: 1 Las desigualdades fundamentales - Toomates · Segunda parte: Desigualdades geométricas. 10 Desigualdades con los lados del triángulo. → Desigualdad Triangular. La Transformación

04

424

2

222222

2222

xzyzxy

xzyzxyzyxxzyzxyzyx

cbacba

10.6

Puesto que son los lados de un triángulo , podemos escribir

zya , xzb , yxc , con 0,, zyx

qpxzyzxyzyxacbcab 3)(3222

qpqpxzyzxyzyxcba 2222 222222

qpacbcab 622

Está claro que acbcabcba 2222 pues qpqp 6222

Veamos la primera desigualdad:

acbcabcba

qpqpqpqpacbcabcba

222

222 0322

Y esta desigualdad se demostró en el bloque teórico.

10.7

a) Aplicando la Transformación de Ravi:

yxa , zyb , xzc

z

yx

yxxzzy

yx

acb

a

2

y de la misma manera

x

zy

bac

b

2

,

y

xz

cba

c

2

3)222(2

1

2

1

2

1

2

1

222

y

x

x

y

y

z

z

y

x

z

z

x

y

x

y

z

x

z

x

y

z

y

z

x

y

xz

x

zy

z

yx

y

xz

x

zy

z

yx

cba

c

bac

b

acb

a

b) De nuevo, mediante la Transformación de Ravi:

32222

xz

y

zy

x

yx

z

xz

y

zy

x

yx

z

c

cba

b

bac

a

acb

Es la Desigualdad de Nesbitt (3.4).

10.8

Ver problema PG/6.15

10.9

Ver problema PG/6.59

10.10

Page 91: 1 Las desigualdades fundamentales - Toomates · Segunda parte: Desigualdades geométricas. 10 Desigualdades con los lados del triángulo. → Desigualdad Triangular. La Transformación

Ver problema PG/7.7

10.11

Ver problema PG/7.8

10.12

Ver Problema PG/7.9

10.13

Podemos suponer, sin pérdida de generalidad, que cba .

Entonces, puesto que estamos trabajando con los lados de un triángulo, se cumple:

cbacbcabacba

En efecto, veamos la segunda desigualdad:

acbcbcbacbcbabaccbcacbab

cbcacbbcabcbacbcab

)()())((0 2222

22

y, efectivamente, cb pues suponemos cb y 0 acb por la desigualdad triangular.

Una vez establecido esto, el problema se resuelve mediante el método de Reordenación,

teniendo en cuenta que cba

cba111

cbacb

bcaba

acbac

cbacc

bcabb

acbaa

111111

Simplificamos la parte de la izquierda:

cbacbabcaacbcbacc

bcabb

acbaa

111

Simplificamos la parte de la derecha:

cb

cacb

a

bbca

c

aabcbac

bbcab

aacba

c

222111

Cancelando términos llegamos a:

b

acc

a

cbb

c

baa

b

cac

a

bcb

c

aba

b

cac

a

bbc

c

aab

)()()(0

)()()(0

222

Multiplicando por abc a ambos lados llegamos a la desigualdad del enunciado:

)()()(0 222 acaccbcbbaba

10.14

La desigualdad de la izquierda:

Page 92: 1 Las desigualdades fundamentales - Toomates · Segunda parte: Desigualdades geométricas. 10 Desigualdades con los lados del triángulo. → Desigualdad Triangular. La Transformación

2)()(3 cbacabcab

está demostrado en el Problema 2.3b. No hace falta que sean lados de un triángulo.

Veamos la desigualdad de la derecha.

Aplicando la Transformación de Ravi, 22 )(4)()(2 zyxcbazyxcba

)(3222 xzyzxyzyxcabcab

Luego

)(3)(2

)(3)(2

)(3)(

)(4)(4)(4)(

222222

2222

22

xzyzxyxzyzxy

xzyzxyzyxxzyzxyzyx

xzyzxyzyxzyx

cabcabzyxcabcabcba

xzyzxy 0 , lo cual es cierto.

10.15

La desigualdad de la izquierda es el problema 2.5a.

Para la desigualdad de la derecha, basta ver que:

)()()()(2 baccabcbaacbcabacbcabcabcab

Y tener en cuenta que acb , bca y cba .

10.16

La desigualdad de la izquierda es la desigualdad de Nesbitt (3.6)

Veamos la desigualdad de la derecha. Observamos que

cba

c

ba

ccbabacbabababa

2

22

Y de la misma manera: cba

b

bc

b

2 y

cba

c

ba

c

2

Por lo tanto

2)(2222

cba

cba

cba

c

cba

b

cba

a

ba

c

ac

b

cb

a

10.17

Basta tener en cuenta que

)()()( 222222222222 cbacbacbacbacbacbacbacba

y aplicar el problema 10.10

10.18

Utilizaremos la siguiente identidad: ))()((

))()((

cacbba

cbcaba

ac

ac

cb

cb

ba

ba

En efecto:

Page 93: 1 Las desigualdades fundamentales - Toomates · Segunda parte: Desigualdades geométricas. 10 Desigualdades con los lados del triángulo. → Desigualdad Triangular. La Transformación

caabbabcaccbcbbaac

bcaccbcaabbacabacb

bcaccbcaabbaaccbba

222222

222222

222222

))()((

))()((

))()((

Y por tanto el numerador de la suma de las tres fracciones es:

))()((222222 cbcabacaabcbbcacba

Luego

(*)))()((

))()((

))()((

))()((

cacbba

cbcaba

cacbba

cbcaba

ac

ac

cb

cb

ba

ba

Ahora utilizamos el Problema 2.3

abccacbba 8))()((

Y por tanto

(**)8

1

8

))()(((*)

a

cb

b

ca

c

ba

abc

cbcaba

Y la equivalencia b de la desigualdad triangular:

1

a

cbcba , 1

b

cacab , 1

c

babac

Luego, finalmente:

8

1111

8

1(**)

10.19

Se deduce directamente del problema 10.14:

32334)(3

)(4)()(3

22

2

cbacba

cabcabcbacabcab

10.20

a)

abbcaacbabbcbaacba

abbcba

acba

abbsas

4422

22))((

Pero

0)(244 22222 cbabaabcabbcaacbab

Pues 22cbacba

Observación: Una manera alternativa:

Page 94: 1 Las desigualdades fundamentales - Toomates · Segunda parte: Desigualdades geométricas. 10 Desigualdades con los lados del triángulo. → Desigualdad Triangular. La Transformación

cbacbabacbabasba

sbasbass

assbsababassbsabbsas

22)(2

)(0)(

0))((

22

22

Lo cual es cierto por la desigualdad triangular.

b) Multiplicando la parte izquierda por 4:

bcacabbcacabbcacabcbabcacab

acbcbacbabcabcaacb

ascscsbsbsas

22

))(())(())((

)22)(22()22)(22()22)(22(

222

Donde hemos utilizado bcacabcba 222 demostrado en el problema 10.15.

11.1

Ver problema PG/6.58

11.2

Ver problema PG/6.72

11.3

Ver problema PG/6.74

11.4

Ver problema PG/6.80

11.5

Ver problema PG/6.85

11.6

Ver problema PG/6.84

12.5

a)

Aplicando La Desigualdad Triangular al triángulo ABD , 2/aADc

Aplicando La Desigualdad Triangular al triángulo ACD , 2/aADb

Luego, sumando las dos desigualdades anteriores,

ADacb

ADacbaADcb

2

22

b) Sea BAD1 , DAC2 , ABD , ACD

Page 95: 1 Las desigualdades fundamentales - Toomates · Segunda parte: Desigualdades geométricas. 10 Desigualdades con los lados del triángulo. → Desigualdad Triangular. La Transformación

Comparando lados y ángulos en ABD : 12

1BDaAD

Comparando lados y ángulos en ACD : 22

1CDaAD

Luego º902

º1802º180 212121 BADBAD .

12.6 Ver problema PG/7.6

12.7

Aplicamos la Desigualdad de Leibniz 12.4:

22229 cbaR

Y aplicando abcABCR 4 (ver GA/11.6.5)

Tenemos:

222

222

2

222222222

2

222222

2

222

2

222222222

34

43

16991616

9

16164

cba

abcABC

ABC

abccba

ABC

cbacbacba

ABC

cbacba

ABC

cba

ABC

cbaRcbaABCRabcABCR

Y ahora, aplicando la desigualdad Cauchy-Schwarz, tenemos

2223 cbacba

Y por tanto

cba

abcABC

cba

abc

cba

abc

cba

abcABC

934

33

3

3334

222222

Page 96: 1 Las desigualdades fundamentales - Toomates · Segunda parte: Desigualdades geométricas. 10 Desigualdades con los lados del triángulo. → Desigualdad Triangular. La Transformación

Tal y como queríamos ver.

12.8

Aplicamos la Desigualdad Cauchy-Schwarz (4.1) a las ternas

crbqap ,, y cba /1,/1,/1

para obtener

(*)111

cbacrbqaprqp

Interpretando geométricamente los valores rqp ,, como alturas de los tres triángulos interiores

que determina P, la expresión crbqap es dos veces su área, y aplicando GA/11.13.1

tenemos:

R

abc

R

abcABCcrbqap

2422

Así pues,

R

abacbc

abc

abacbc

R

abc

abc

abacbc

R

abc

222(*)

Y por último solo nos queda aplicar el Problema 2.5a 222 cbaabacbc

para llegar al resultado deseado.

Fuente de esta solución: web "El blog de Leo" blog.nekomath.com

12.9

a) El punto P divide el triángulo en tres triángulos de alturas PD, PE, y PF, con bases todas

iguales al lado a , luego

PFPEPDaaPFaPEaPD

ABC 2222

Pero, por otro lado, 2

haABC

De donde se deduce directamente que hPFPEPD

b) Utilizamos 4.4 y el Lema de Viviani demostrado en el apartado a:

(*)99111

9111

hPFPEPDPFPEPDPFPEPDPFPEPD

Por otro lado, por Pitágoras, 2

3

4

3

2

22

22 ah

aaah

aa

36

2/3

9(*)

Tal y como queríamos ver.

Page 97: 1 Las desigualdades fundamentales - Toomates · Segunda parte: Desigualdades geométricas. 10 Desigualdades con los lados del triángulo. → Desigualdad Triangular. La Transformación

b) De nuevo, aplicando 4.4,

aahPDPFPFPEPEPD

PDPFPFPEPEPDh

PDPFPFPEPEPDPDPFPFPEPEPD

33

2/32

9

2

9111

9111

2

9111

12.10

a)

ABC

a

hh

a

ABChaABC

a

aa

2

12

2

ABCrs

rABC

scba

ABCABC

c

ABC

b

ABC

a

hhh cba

1

2

1

222

111

Lo cual es cierto por GA/11.4.8.

Observación: Una manera más elegante podría ser tener en cuenta que, si I es el incentro del

triángulo:

ABC

IBC

ah

ar

h

r

aa

, y de la misma forma

ABC

ICA

h

r

b

y

ABC

IAB

h

r

c

Y por tanto

1

ABC

ABC

ABC

IAB

ABC

ICA

ABC

IBC

h

r

h

r

h

r

cba

b) Aplicamos 4.4 y el apartado a:

rhhhrh

r

h

r

h

rhhh

hhhhhh

cba

cba

cba

cba

cba

99

93111 2

12.11

a) Aplicaremos que las áreas de triágulos con la misma base son proporcionales a las alturas

correspondientes. En nuestro caso:

93111

111

2

AHBAHCBHCAHBAHCBHC

AHBAHCBHCABC

AHB

ABC

AHC

ABC

BHC

ABC

HF

CF

HE

BE

HD

AD

En donde hemos aplicado la desigualdad 4.4.

Page 98: 1 Las desigualdades fundamentales - Toomates · Segunda parte: Desigualdades geométricas. 10 Desigualdades con los lados del triángulo. → Desigualdad Triangular. La Transformación

b) Sean HABSHACSHBCSABCS 321 ,,,

32

1

1

1

SS

S

SS

S

HDDA

HD

HA

HD

, y de la misma forma

31

2

SS

S

HB

HE

y

21

3

SS

S

HC

HF

y por tanto:

2

3

21

3

31

2

32

1

SS

S

SS

S

SS

S

HC

HF

HB

HE

HA

HD

Es una aplicación directa de la Desigualdad de Nesbitt (3.6).

Fuente de la solución: Inequalities A Mathematical Olympiad Approach (Radmila Bulajich Manfrino) pág. 62.

12.12

Sea ACa , CEb y AEc .

Aplicamos la Desigualdad de Ptolomeo 12.2 al cuadrilátero ACEF :

cFCAFbFEa

Puesto que se cumple FEFA , tenemos

FC

FA

ba

ccFCbaFA

)(

Y de la misma manera:

BE

BC

cb

a

y

DA

DE

ac

b

Y por tanto:

2

3

ba

c

ac

b

cb

a

FC

FA

DA

DE

BE

BC

Que es la desigualdad de Nesbitt (3.6).

12.13

En primer lugar vamos a estudiar uno de los segmentos por separado.

Consideremos el segmento AP.

Sea A' el punto medio del lado BC y P' el punto medio del arco BC.

Sean D y D' las respectivas proyecciones de A y P en el segmento BC.

Page 99: 1 Las desigualdades fundamentales - Toomates · Segunda parte: Desigualdades geométricas. 10 Desigualdades con los lados del triángulo. → Desigualdad Triangular. La Transformación

Por semejanza de triángulos, ''' PA

AD

PD

AD

LP

AL

Luego el valor mínimo se obtiene cuando 'PP , es decir, cuando AP es la bisectriz del ángulo

A (ver GA/11.12.1).

Así pues, a partir de ahora supondremos que AP, BQ y CR son las bisectrices respectivas de los

ángulos A, B y C.

Pero en este caso sabemos (ver GA/11.4.4) que

cb

acBL

,

cb

abLC

y

2

2 1cb

abcAL

Y por tanto:

2

22

2

2

22

2211

a

acb

cb

bca

cb

abc

cb

ab

cb

ac

cb

abc

LCBL

AL

LPAL

AL

LP

AL

Y de la misma manera, con los otros dos segmentos del enunciado tenemos:

2

22)(

b

bac

MQ

BM ,

2

22)(

c

cba

NR

CN

Y por tanto, aplicado la desigualdad del Problema 2.5b:

(*)3

3

13

)()(

)()(

2

2

2

2

2

2

2

2

22

2

22

2

22

c

ba

b

ac

a

cb

c

ba

b

ac

a

cb

c

cba

b

bac

a

acb

NR

CN

MQ

BM

LP

AL

Y finalmente, teniendo en cuenta que

6222

a

c

c

a

c

b

b

c

b

a

a

b

c

b

c

a

b

a

b

c

a

c

a

b

c

ba

b

ac

a

cb

Llegamos a

Page 100: 1 Las desigualdades fundamentales - Toomates · Segunda parte: Desigualdades geométricas. 10 Desigualdades con los lados del triángulo. → Desigualdad Triangular. La Transformación

9363

1(*) 2

Observamos que la igualdad se cumple si y solo si cba .

Observación: Otra manera de acabar este problema sería la siguiente:

933222232

3)()(

2222

2

2

2

2

2

2

2

2

2

2

2

2

2

2

2

2

2

b

ca

a

bc

c

ab

c

a

a

c

b

c

c

b

a

b

b

a

c

ba

b

ac

a

cb

En donde hemos tenido en cuenta que

22

2

2

2

a

b

b

a y 333

222222

cba

abbcca

b

ca

a

bc

c

ab

Fuente de esta solución: Inequalities A Mathematical Olympiad Approach (Radmila Bulajich Manfrino), pág. 63

12.14

Nos basamos en que las áreas de triángulo compartiendo una misma base son proporcionales a

las alturas:

93111

111

2

APBCPABPCAPBCPABPC

APBCPABPCABC

APB

ABC

CPA

ABC

BPC

ABC

PS

CF

PR

BE

PQ

AD

En donde hemos utilizado que APBCPABPCABC y la desigualdad 4.4.

12.15

Si P es el baricentro del triángulo, basta aplicar GA/11.5.3b:

6222 PN

CP

PM

BP

PL

AP

Veamos el recíproco. Supongamos que 6PN

CP

PM

BP

PL

AP.

Entonces:

PN

CN

PM

BM

PL

AL

PN

PNCP

PM

PMBP

PL

PLAP

PN

PN

PN

CP

PM

PM

PM

BP

PL

PL

PL

AP

PN

CP

PM

BP

PL

AP

96

Pero sabemos, por GA/9.2.3, que 1CN

PN

AM

BM

AL

PL

Page 101: 1 Las desigualdades fundamentales - Toomates · Segunda parte: Desigualdades geométricas. 10 Desigualdades con los lados del triángulo. → Desigualdad Triangular. La Transformación

Y por la desigualdad 4.4 que

9

PN

CN

PM

BM

PL

AL

CN

PN

AM

BM

AL

PL

Por lo anterior, esta desigualdad es igualdad, y esto solo sucede cuando

3PN

CN

PM

BM

PL

AL

Y esto solo sucede cuando P es el baricentro (faltaría por demostrar esto último).

12.16

a) Sea H el ortocentro del triángulo. Por GA/11.6.9 sabemos que HD=DD', HE=EE', HF=FF',

luego

HF

CF

HE

BE

HD

AD

FF

CF

EE

BE

DD

AD

'''

Por GA/9.2.3 sabemos que

1CF

HF

BE

HE

AD

HD

Aplicando la Desigualdad 4.4 tenemos

9

HF

CF

HE

BE

HD

AD

CF

HF

BE

HE

AD

HD

Luego 9HF

CF

HE

BE

HD

AD, tal y como queríamos ver.

b)

4'''

3'

1'

1'

1

''''''

CF

FF

BE

EE

AD

DD

CF

FF

BE

EE

AD

DD

CF

FFCF

BE

EEBE

AD

DDAD

CF

CF

BE

BE

AD

AD

Puesto que ya hemos visto en el apartado anterior que 1'''

CF

FF

BE

EE

AD

DD.

Aplicando la Desigualdad 4.4 tenemos

4

9

'''9

'''4

9'''

'''

CF

CF

BE

BE

AD

AD

CF

CF

BE

BE

AD

AD

CF

CF

BE

BE

AD

AD

CF

CF

BE

BE

AD

AD

tal y como queríamos ver.

12.17

a) Por GA/11.4.4d sabemos que

cb

asbcsla

)(42 , ca

bsacslb

)(42 , ba

csabslc

)(42

Page 102: 1 Las desigualdades fundamentales - Toomates · Segunda parte: Desigualdades geométricas. 10 Desigualdades con los lados del triángulo. → Desigualdad Triangular. La Transformación

y por tanto:

))()((

4

))()((

)(4

))()((

4

))()((

))()((4)(4)(4)(4

242223222223222223

32223222

bacacb

rscba

bacacb

rsscba

bacacb

ABCscba

bacacb

csbsasscba

ba

csabs

ca

bsacs

cb

asbcslll cba

Donde hemos aplicado la Fórmula de Heron (GA/10.5.11) y GA/11.4.8. Luego:

rsrsbacacb

abclll cba

22

))()((

8

Donde hemos aplicado la desigualdad del problema 2.3.

b) Puesto que 2)(4 cbbc (Desigualdad AM-GM) tenemos que )()(42 ass

cb

asbcsla

, y

de la misma forma

)(2 bsslb y )(2 csslc .

y por tanto:

2

222

)(2

3

)()()()(

scbacba

s

csbsasscssbssasslll cba

c) Basta aplicar el Problema 2.5a, o de una manera directa:

22

2

2))(())(())((222

cs

bass

bsassbsassbsassllbsassll baba

En donde hemos aplicado la desiguadad AM-GM, y por tanto:

2

2222s

cbas

bs

as

csllllll accbba

12.18

Puesto que no hay relación entre los puntos M, N y P, vamos a demostrar que

RAM

bc2 , R

BN

ac2 y R

CP

ab2

Veamos la primera, las otras dos se demuestran con argumentos similares.

El punto de una recta más próximo a un punto dado se encuentra trazando la perpendicular por

dicho punto, así pues

ahAM , con ah la altura del triángulo correspondiente al vértice A.

Luego

Rh

bc

AM

bc

a

2

Page 103: 1 Las desigualdades fundamentales - Toomates · Segunda parte: Desigualdades geométricas. 10 Desigualdades con los lados del triángulo. → Desigualdad Triangular. La Transformación

Esta última igualdad se puede demostrar por, por ejemplo:

aaa hRbc

haRabcABCRabc

haABC 2

244

2

Una manera alternativa de resolver este problema podría ser tener en cuenta la observación en

GA/9.2.2

)sin(22

)sin(

4AMBR

AM

bcAMBAMaABC

R

abc

Y de la misma forma )sin(2 BNARBN

ca y )sin(2 APCR

CP

ab

Y por tanto RRPCBNAAMBRCP

ab

BN

ca

AM

bc632)sin()sin()sin(2

12.19

Primera versión.

Aplicando la desigualdad 4.4

acbcabacbcabacbcab

acbcab

91113

111 2

Es suficiente demostrar

acbcabRRacbcab

2

29

19

Pero aplicando la desigualdad del problema 2.5a y la desigualdad de Leibniz (12.4) 2222 9Rcbaacbcab

Tal y como queríamos ver.

Segunda versión.

Aplicamos las identidades conocidas R

abcABC

4 ,

2

)( rcbasrABC

Y la desigualdad de Euler rR 2

2

1

2

1

4

/2111

RrRABCR

rABC

abc

bac

acbcab

13.1

Esta inecuación lleva implícita la condición 2/112021 xxx , y también

0x , pues 0x anularía el denominador de la división de la parte izquierda.

Multiplicamos y dividimos la parte izquierda por el conjugado para eliminar la raíz:

2

2

22

22

22

2

2

2222

2

2114

2114

211211

2114

211

4

42211211

2211)21(1211211

xx

xx

xx

xx

x

x

xxxx

xxxxx

Luego la inecuación del enunciado es equivalente a:

Page 104: 1 Las desigualdades fundamentales - Toomates · Segunda parte: Desigualdades geométricas. 10 Desigualdades con los lados del triángulo. → Desigualdad Triangular. La Transformación

8/454/4514/4924/492/721

2/7217212

9221212192211

2

2

xxx

xx

xxxxx

Así pues, el resultado es 8

45

2

1

x y 0x

13.2

Esta inecuación lleva implícitas dos condiciones:

31101

303

x

xx

xx

Tomando 13)( xxxf , vemos que esta función es decreciente en el intervalo

31 x , y que toma valores desde 2)1( f hasta 2)3( f . Luego existirá un único

valor a tal que 2/1)( af , y el intervalo buscado será ax 1 .

Observamos que 0)1( f , luego 1a .

El problema se reduce a resolver la ecuación 2

113 aa , que elevando al cuadrado se

convierte en la ecuación de segundo grado 064

3322 aa , cuyas raíces son

8

311a .

Puesto que sabemos que 1a , la única solución válida es 8

311a y la solución del

problema es 8

3111 x .

Fuente de esta solución: International Mathematical Olympiads 1959-1977 Compiled and with solutions by Samuel L.Greitzer, pág. 46.

Nota: Si intentamos resolver este mismo problema por métodos digamos más convencionales,

vemos que aparece en el resultado un intervalo no aceptable:

0264

33

8

1523

8

1523

8

1513

8

1513

4

15132132

4

151324

4

1

13213132

113

2

1

2

2

2

2

2

2

22

xxxx

xxxxxx

xxxxxx

xxxxxxxx

Para resolver esta última inecuación determinamos las raíces de la ecuación cuadrática:

8

31102

64

33 2 xxx

Page 105: 1 Las desigualdades fundamentales - Toomates · Segunda parte: Desigualdades geométricas. 10 Desigualdades con los lados del triángulo. → Desigualdad Triangular. La Transformación

Luego 8

311

8

31102

64

33 2 xxxx

Sin embargo, el intervalo 8

311x no es solución de la inecuación original.

13.3

Dominio de definición: La inecuación no está definida en bx y b

x1

.

01

10

1

10

1

01

110

1

1

1

1

2222

bxbx

x

bxbx

xba

bxbx

babxax

bxbx

axbxbxax

bx

ax

bx

ax

bx

ax

bx

ax

En donde hemos tenido en cuenta que 1 ba . Dividir entre b no cambia el signo, y por tanto:

0

1

)1)(1(0

1

12

bxb

x

xx

bxbx

x

Se cumplirá cuando el numerador y el denominador tengan el mismo signo. El numerador es

negativo en 11 x , y el denominador es negativo en bxb /1 .

El conjunto solución es ,1/1,1, bb .

14.1

Si 2,, cba ,

12

1111

4

1

2

14262

4

11

4

11

4

11

cbaaccbba

cbacba

ac

cb

ba

Y la igualdad solo puede darse si 2 cba , y se comprueba que este caso es solución de la

ecuación.

Page 106: 1 Las desigualdades fundamentales - Toomates · Segunda parte: Desigualdades geométricas. 10 Desigualdades con los lados del triángulo. → Desigualdad Triangular. La Transformación

Supongamos ahora que al menos una de las tres incógnitas es menor que 2. Podemos suponer,

sin pérdida de generalidad, que 1a .

La ecuación nos queda

11

1

1

11

1

1

cbccbb

De nuevo, si 3, cb ,

11

1

1

11

1

1

4

1

5

1

1

1

4

1

1

1

4

1

6

11

4

1

1

1

cbccbb

cb

c

cb

b

. Luego 2,1 cb

El caso 1 cb no satisface la ecuación: 11

1

2

1

2

1

2

1

El caso 1b , 2c no satisface la ecuación: 12

1

3

1

3

1

2

1

El caso 1c , 2b tampoco por simetría.

Finalmente, el caso 2 cb tampoco satisface la ecuación: 13

1

3

1

4

1

3

1

Así pues, el único caso aceptable es 2 cba

14.2

Primera parte: Demostración de la desigualdad.

Supongamos que no es cierto, es decir, que existen nba ,, enteros positivos tales que ba y

21 nab , pero 34 nba

22222 1214414344344)()( nnnnnabnabbaba

Luego

nbanba 212

Por otro lado, por la desigualdad AM-GM:

2

2

baab

Y por tanto:

2

22

2

2

2

21 n

nbaabn

llegando a contradicción.

Segunda parte. Resolución de la igualdad.

De nuevo utilizamos identidad abbaba 4)()( 22 .

121214414344)()(22222 nbannnnnabbaba .

Page 107: 1 Las desigualdades fundamentales - Toomates · Segunda parte: Desigualdades geométricas. 10 Desigualdades con los lados del triángulo. → Desigualdad Triangular. La Transformación

La ecuación 34 nba implica que 34 n sea un cuadrado perfecto impar, es decir,

21234 un para cierto u entero no negativo, y 1212342

uunba .

144441441234 2222 uunuunuuun

1121212212

1222

uuuuunbununb

uba

nba

221121212 22 uuuubuauba

Luego el conjunto solución son todos los números nba ,, tales que

12

1

1

22

2

2

2

uba

uun

ub

uua

para todo entero u positivo.

En efecto, se cumple:

111

22322222122

222

234223422

nuu

uuuuuuuuuuuuab

Fuente de esta solución: Soluciones oficiales OME.

Page 108: 1 Las desigualdades fundamentales - Toomates · Segunda parte: Desigualdades geométricas. 10 Desigualdades con los lados del triángulo. → Desigualdad Triangular. La Transformación

Fuentes.

2.20 Basics of Olympiad Inequalities Samin Riasat (2008) 4

2.21 Basics of Olympiad Inequalities Samin Riasat (2008) 4

2.22 "2010 IMO Summer IMO Training: Inequalities Adrian Tang". 2

2.23 Mathematical Excalibur Volume 5, Number 4, September 2000 – November 2000 3

1.24 Secrets in Inequalities (volume 1) (Pham Kim Hung) 17

4.12 "2010 IMO Summer IMO Training: Inequalities Adrian Tang". 3

4.13 Secrets in Inequalities (volume 1) (Pham Kim Hung) 34

4.14 Secrets in Inequalities (volume 1) (Pham Kim Hung) 36

5.1 Inequalities A Mathematical Olympiad Approach (Radmila Bulajich Manfrino) 4

5.2 Inequalities A Mathematical Olympiad Approach (Radmila Bulajich Manfrino) 4

5.3 Inequalities A Mathematical Olympiad Approach (Radmila Bulajich Manfrino) 4

5.9 Secrets in Inequalities (volume 1) (Pham Kim Hung) 92

5.11 Secrets in Inequalities (volume 1) (Pham Kim Hung) 54

6.5 "2010 IMO Summer IMO Training: Inequalities Adrian Tang". 3

6.7 Mathematical Excalibur Volume 5, Number 4, September 2000 – November 2000 2

6.8 Secrets in Inequalities (volume 1) (Pham Kim Hung) 70

7.7 https://artofproblemsolving.com/wiki/index.php?title=Homogenization&oldid=78488

8.1 103 Trigonometry Problems From the Training… (Titu Andreescu…) 65

8.2 "Contest Problem Book V" 158

8.10 101 Problems in Algebra from the training of the USA IMO team (Adreescu, Feng) 2

9.1 Problem-Solving Strategies (Arthur Engels, 1998) 165

9.2 103 Trigonometry Problems From the Training… (Titu Andreescu…) 65

9.3 103 Trigonometry Problems From the Training… (Titu Andreescu…) 65

9.4 Problem-Solving Strategies (Arthur Engels, 1998) 174

9.5 Compiled and Solved Problems in Geometry and Trigonom... (Florentin Smarandache) 41

9.4 "AIME Solution Set 2015 David Altizio January 12, 2018" 12

9.4 103 Trigonometry Problems From the Training… (Titu Andreescu…) 65

10.9 "Contest Problem Book V" 170

10.10 Inequalities A Mathematical Olympiad Approach (Radmila Bulajich Manfrino) 15

10.11 Inequalities A Mathematical Olympiad Approach (Radmila Bulajich Manfrino) 54

10.12 Inequalities A Mathematical Olympiad Approach (Radmila Bulajich Manfrino) 56

10.14 Inequalities A Mathematical Olympiad Approach (Radmila Bulajich Manfrino) 57

10.15 Inequalities A Mathematical Olympiad Approach (Radmila Bulajich Manfrino) 57

11.1 103 Trigonometry Problems From the Training… (Titu Andreescu…) 66

11.2 103 Trigonometry Problems From the Training… (Titu Andreescu…) 64 y 68

12.5 International Mathematical Olympiads 1959-1977 Samuel L.Greitzer 3

12.8 web "El blog de Leo" blog.nekomath.com

12.9 Inequalities A Mathematical Olympiad Approach (Radmila Bulajich Manfrino) 61

12.10 Inequalities A Mathematical Olympiad Approach (Radmila Bulajich Manfrino) 62

12.11 Inequalities A Mathematical Olympiad Approach (Radmila Bulajich Manfrino) 62

12.13 Inequalities A Mathematical Olympiad Approach (Radmila Bulajich Manfrino) 63

12.14 Inequalities A Mathematical Olympiad Approach (Radmila Bulajich Manfrino) 65

13.1 International Mathematical Olympiads 1959-1977 Samuel L.Greitzer 46

Fuente principal del capítulo 6: Secrets in Inequalities (volume 1) (Pham Kim Hung), páginas 67 en adelante.